2015
l
2016 FREE
Preparing for the ACT
®
Test
www.actstudent.org
What’s Inside
Full-Length Practice Tests, including a Writing Test
Information about the Optional Writing Test
Strategies to Prepare for the Tests
What to Expect on Test Day
Esta publicación también se puede ver o descargar
en español en www.actstudent.org
Contents
1. General Preparation for the ACT Tests ............ 2
2. Strategies for Taking the ACT Tests............... 4
3. What to Expect on Test Day..................... 9
4. Taking the Practice Tests...................... 10
Practice Multiple-Choice Tests............... 11
Practice Writing Test ...................... 53
5. Scoring Your Tests ........................... 56
How to Score the Multiple-Choice Tests ....... 56
How to Score the Writing Test ............... 61
6. Sample Answer Document..................... 63
A Message to Students
This booklet, which is provided free of charge, is intended
to help you do your best on the ACT
®
test. Included in
this booklet are complete practice tests—“retired” ACT
questions that were administered to students on a National
test date—including a writing prompt, a sample answer
document, answer keys, and self-scoring instructions.
Read this booklet carefully and take the practice tests well
before test day so you will be familiar with the tests, what
they measure, and the strategies you can use to do your
best on test day.
Go to www.actstudent.org for additional ACT test
preparation materials, including ACT Online Prep
, The
Real ACT Prep Guide, sample questions, and the Question
of the Day.
The ACT is administered nationally and internationally
to examinees in English, including all instructions and
questions. Select states testing as part of the State and
District testing program permit the use of translated
instructions, but such testing does not result in a college-
reportable score.
ACT is committed to representing the diversity of society
in all its aspects, including race, ethnicity, and gender.
Thus, test passages, questions, and writing prompts are
deliberately chosen to reflect a range of cultures.
ACT is also committed to ensuring that test questions and
writing prompts are fair—that they do not disadvantage
any particular group of examinees. Extensive reviews of
the fairness of test materials are rigorously conducted by
both ACT staff and external consultants. ACT also employs
statistical procedures to help ensure that test materials do
not unfairly affect the performance of any group.
Note: Since the ACT is a curriculum-based achievement
test, research is periodically conducted and tests are
updated accordingly to ensure test content continues
to reflect classroom instruction and remains a relevant
predictor of college and career readiness. As a result, you
may notice subtle differences between this practice test
and the test you actually take on test day.
1
General Preparation
for the ACT Tests
General Test-Taking
Strategies for the ACT
The ACT contains multiple-choice tests in four areas:
English, mathematics, reading, and science. Each of these
tests contains questions that offer either four or five answer
choices from which you are to choose the correct, or best,
answer. The following suggestions apply to all four tests:
Pace yourself.
The time limits set for each test give nearly everyone enough
time to finish all the questions. However, because the
English, reading, and science tests contain a considerable
amount of text, it is important to pace yourself so you will not
spend too much time on one passage. Similarly, try not to
spend too much time puzzling over an answer to a specific
problem in the mathematics test. Go on to the other
questions and come back if there is time. Your supervisor
will announce when you have five minutes remaining on
each test.
Read the directions for each test carefully.
Before you begin taking one of the tests, read the
directions carefully. The English, reading, and science
tests ask for the “best” answer. Do not respond as soon as
you identify a correct answer. Read and consider all of the
answer choices and choose the answer that best responds
to the question.
The mathematics test asks for the “correct” answer. Read
each question carefully to make sure you understand the
type of answer required. Then, you may want to work out
the answer you feel is correct and look for it among the
choices given. If your answer is not among the choices
provided, reread the question and consider all of the
answer choices.
Read each question carefully.
It is important that you understand what each question
asks. Some questions will require you to go through several
steps to find the correct or best answer, while others can
be answered more quickly.
Answer the easy questions rst.
The best strategy for taking the tests is to answer the easy
questions and skip the questions you find difficult. After
answering all of the easy questions, go back and answer
the more difficult questions if you have time.
2
© 2015 by ACT, Inc. All rights reserved.
NOTE: This booklet is covered by federal copyright laws that prohibit the
reproduction of the test questions without the prior express, written per-
mission of ACT, Inc. No portion of this booklet may be copied or distrib-
uted without written permission of ACT.
ACT endorses the Code of Fair Testing Practices in Education and the Code of
Professional Responsibilities in Educational Measurement, guides to the conduct
of those involved in educational testing. ACT is committed to ensuring that each
of its testing programs upholds the guidelines in each Code. A copy of each
Code may be obtained free of charge from ACT Customer Services (70), PO Box
1008, Iowa City, IA 52243-1008, 319.337.1429.
Use logic on more difficult questions.
When you return to the more difficult questions, try to
use logic to eliminate incorrect answers to a question.
Compare the answer choices to each other and note how
they differ. Such differences may provide clues as to what
the question requires. Eliminate as many incorrect answers
as you can, then make an educated guess from the
remaining answers.
Answer every question.
Your score on the tests will be based only on the number
of questions that you answer correctly; there is no penalty
for guessing. Thus, you should answer every question
within the time allowed for each test.
Review your work.
If there is time left after you have answered every question in
a test, go back and check your work on that test. You will not
be allowed to go back to any other test or mark responses to
a test after time has been called on that test.
Be precise in marking your answer document.
Be sure that you properly fill in the correct ovals on your
answer document. Check to be sure that the number of
the line of ovals on your answer document is the same as
the number of the question you are answering and that you
mark only one response for each question.
Erase completely.
If you want to change a multiple-choice answer, be sure
to use a soft eraser that will not leave smudges and erase
the unintended mark completely. Do not cross out answers
or use correction fluid or tape; you must erase. Correction
fluid/tape, smudges, or unintended marks may cause
errors in scoring.
To students approved to test at National test centers
with extended time:
You will be allowed up to 5 hours total to work on the
multiple-choice tests at your own pace, including breaks
between tests. If you are taking the ACT with writing, you
will be allowed up to 6 hours total to work on all five tests.
General Test-Taking Strategies
for the ACT Writing Test
The ACT writing test lets you show your skill in composing
an essay. It measures writing proficiencies that are taught
in high school and are important for readiness to succeed
in entry-level college composition courses.
The following general strategies will help if you take the
ACT writing test.
Pace yourself.
You will have 40 minutes to write your essay. It is important
to pace yourself in the way that best suits your personal
writing strategy. Many writers do best when they spend
part of their time planning the essay, most of their time
writing the essay, and the last part of their time reviewing
the essay to make corrections and revisions. Budget your
time based on your experience in taking essay tests in
school and in other circumstances when you’ve done
writing within a time limit. Your supervisor will announce
when you have five minutes remaining on the writing test.
Read the directions carefully.
In writing your essay, you will be expected to engage
meaningfully with the issue and perspectives presented by
the prompt. Before you begin to plan and write, read and
consider all prompt material carefully.
Read the writing prompt carefully.
It is important that you understand exactly what the
writing prompt asks you to do. Be sure you have a clear
understanding of the issue in the writing prompt and of the
question you must respond to before you start to plan and
write your essay.
Write (or print) legibly in the answer folder.
If your readers cannot read what you have written, they will
not be able to score your essay. You must write your essay
using a soft lead No. 2 pencil (not a mechanical pencil
or ink pen) on the lined pages in the answer folder. You
may not need all the lined pages, but to ensure you have
enough room to finish, do not skip lines.
Make corrections clear.
If you make corrections, do so thoroughly and legibly. You
may write corrections or additions neatly between the lines
of your essay, but do not write in the margins.
Preparing for Test Day
Prepare well in advance for the tests.
Know what to expect on test day. Familiarize yourself
with the information in this booklet, and at
www.actstudent.org.
Most procedures in this booklet refer to testing on a
National or International test date at an ACT test center.
Procedures may differ slightly if you test at another
location.
Take the practice tests in order and review your
responses.
Get plenty of rest the night before the tests.
Carefully review the “Test Day Checklist” at
www.actstudent.org.
Bring the following items with you to the test center:
1. Your paper ticket (if you test on a National or
International ACT test date). You will not be
admitted to test without it.
2. Acceptable photo identification. See details on
your ticket or at www.actstudent.org. If you do
not present acceptable photo identification with
your ticket at check-in, you will not be admitted to
test.
3. Sharpened soft lead No. 2 pencils and good
erasers (no mechanical pencils or ink pens). Do
not bring any other writing instruments; you will
not be allowed to use them.
3
4. A watch to pace yourself. Do not bring a watch
with an alarm, because it will disturb other
students. If your alarm sounds during testing, you
will be dismissed and your answer document will
not be scored. Your supervisor will announce when
you have five minutes remaining on each test.
5. A permitted calculator may be used on the
mathematics test only. It is your responsibility
to know whether your calculator is permitted.
For the most current information on the ACT
calculator policy, visit www.actstudent.org or call
800.498.6481 for a recorded message.
2
Strategies for Taking
the ACT Tests
The ACT measures the knowledge, understanding, and
skills that you have acquired throughout your education.
Although the sum total of what a person has learned cannot
be changed, your performance in a specific area can be
affected by adequate preparation, especially if it has been
some time since you have taken a course in that area.
There are three strategies that can help you to prepare
yourself for the content included in the ACT:
Familiarize yourself with the content of the ACT tests.
Review the information about the tests that is provided on
the following pages. Note which content areas make up a
large proportion of the tests and which do not. The specific
topics included in each content area are examples of
possible topics; they do not include all of the possibilities.
Refresh your knowledge and skills in the content areas.
Review those content areas you have studied but are
not fresh in your mind. Spend your time refreshing your
knowledge and skills in the content areas that make up
large portions of the tests.
Identify the content areas you have not studied.
If unfamiliar content areas make up major portions of
the tests, consider taking coursework to help you gain
knowledge and skills in these areas before you take the
ACT. Because the ACT measures knowledge and skills
acquired over a period of time, it is unlikely that a “cram”
course covering material that is unfamiliar to you will help
you improve your scores. Longer-term survey courses will
be most helpful to you, because they aim to improve your
knowledge through sustained learning and practice.
ACT English Test
The ACT English test is a 75-question, 45-minute test
that measures your understanding of the conventions of
standard written English (punctuation, grammar and usage,
and sentence structure) and of rhetorical skills (strategy,
organization, and style). Spelling, vocabulary, and rote
recall of rules of grammar are not tested. The test consists
of five essays, or passages, each of which is accompanied
by a sequence of multiple-choice test questions. Different
passage types are employed to provide a variety of
rhetorical situations. Passages are chosen not only for
their appro priateness in assessing writing skills but also to
reflect students’ interests and experiences.
Some questions refer to underlined portions of the passage
and offer several alternatives to the underlined portion.
You must decide which choice is most appropriate in
the context of the passage. Some questions ask about
an underlined portion, a section of the passage, or the
passage as a whole. You must decide which choice best
answers the question posed. Many questions offer “NO
CHANGE” to the passage as one of the choices. The
questions are numbered consecutively. Each question
number refers to a correspondingly numbered portion
underlined in the passage or to a corresponding numeral in
a box located at the appropriate point in the passage.
Three scores are reported for the ACT English test: a total
test score based on all 75 questions, a subscore in Usage/
Mechanics based on 40 questions, and a subscore in
Rhetorical Skills based on 35 questions.
Tips for Taking the ACT English Test
Pace yourself.
The ACT English test contains 75 questions to be completed
in 45 minutes. If you spend 1
1
2 minutes skimming through
each passage before responding to the questions, then you
will have 30 seconds to answer each question. If possible,
spend less time on each question and use the remaining
time allowed for this test to review your work and return to the
questions on this test that were most difficult for you.
Be aware of the writing style used in each passage.
The five passages cover a variety of topics and are written
in a variety of styles. It is important that you take into account
the writing style used in each passage when you respond
to the questions. In responding to a question, be sure to
understand the context of the question. Consider how the
sentence containing an underlined portion fits in with the
surrounding sentences and into the passage as a whole.
Examine the underlined portions of the passage.
Before responding to a question with an underlined portion,
carefully examine what is underlined in the text. Consider
the elements of writing that are included in each underlined
portion. Some questions will ask you to base your decision
on some specific element of writing, such as the tone or
emphasis the text should convey. Some questions will ask
you to choose the alternative to the underlined portion
that is NOT or LEAST acceptable. The answer choices for
each question will contain changes in one or more of those
elements of writing.
Be aware of questions with no underlined portions.
You will be asked some questions about a section of the
passage or about the passage as a whole, in light of a
given rhetorical situation. Questions of this type are often
identified by a question number in a box located at the
appropriate point in the passage. Questions about the
entire passage are placed at the end of the passage and
introduced by a horizontal box enclosing the following
instruction: “Questions ___ and ___ ask about the
preceding passage as a whole.”
Note the differences in the answer choices.
Many of the questions in the test will involve more than one
aspect of writing. Examine each answer choice and how it
differs from the others. Be careful not to select an answer
that corrects one error but causes a different error.
4
Determine the best answer.
Two approaches can be taken to determine the best answer
to a question in which you are to choose the best alternative
to an underlined portion. In the first approach, you can
reread the sentence or sentences, substituting each of
the possible answer choices for the underlined portion to
determine the best choice. In the second approach, you can
decide how the underlined portion might best be phrased in
standard written English or in terms of the particular question
posed. If you think the underlined portion is the best answer,
you should select “NO CHANGE.” If not, you should check
to see whether your phrasing is one of the other answer
choices. If you do not find your phrasing, you should choose
the best of the answers presented. For questions cued by
a number in a box, you must decide which choice is most
appropriate in terms of the question posed or the stated
rhetorical situation.
Reread the sentence, using your selected answer.
Once you have selected the answer you feel is best, reread
the corresponding sentence(s) of the passage, inserting
your selected answer at the appropriate place in the text
to make sure it is the best answer within the context of the
passage.
Content Covered by the ACT English Test
Six elements of effective writing are included in the
English test: punctuation, grammar and usage, sentence
structure, strategy, organization, and style. The questions
covering punctuation, grammar and usage, and sentence
structure make up the Usage/Mechanics subscore. The
questions covering strat egy, organization, and style make
up the Rhetorical Skills subscore. A brief description and
the approximate percentage of the test devoted to each
element of effective writing are given below.
USAGE/MECHANICS
Punctuation (10–15%). Questions in this category test
your knowledge of the conventions of internal and end-of-
sentence punctuation, with emphasis on the relationship of
punctuation to meaning (for example, avoiding ambiguity,
indicating appositives).
Grammar and Usage (15–20%). Questions in this cate gory
test your understanding of agreement between subject
and verb, between pronoun and antecedent, and between
modifiers and the word modified; verb formation; pronoun
case; formation of comparative and superlative adjectives
and adverbs; and idiomatic usage.
Sentence Structure (20–25%). Questions in this category
testyour understanding of relationships between
andamong clauses, placement of modifiers, and shifts in
construction.
RHETORICAL SKILLS
Strategy (15–20%). Questions in this category test how
well you develop a given topic by choosing expressions
appropriate to an essay’s audience and purpose; judging
the effect of adding, revising, or deleting supporting
material; and judging the relevancy of statements in
context.
Organization (10–15%). Questions in this category test how
well you organize ideas and choose effective opening,
transitional, and closing sentences.
Style (15–20%). Questions in this category test how well
you choose precise and appropriate words and images,
maintain the level of style and tone in an essay, manage
sentence elements for rhetorical effectiveness, and
avoid ambiguous pronoun references, wordiness, and
redundancy.
ACT Mathematics Test
You may use a calculator on the mathematics test.
Seewww.actstudent.org for details about prohibited
models and features.
The ACT mathematics test is a 60-question, 60-minute test
designed to assess the mathematical skills students have
typically acquired in courses taken up to the beginning
of grade 12. The test presents multiple-choice questions
that require you to use reasoning skills to solve practical
problems in mathematics. Most questions are self-
contained. Some questions may belong to a set of several
questions (e.g., several questions about the same graph
or chart). Knowledge of basic formulas and computational
skills are assumed as background for the problems, but
recall of complex formulas and extensive computation is
not required. The material covered on the test emphasizes
the major content areas that are prerequisites to successful
performance in entry-level courses in college mathematics.
Four scores are reported for the ACT mathematics test:
a total test score based on all 60 questions, a subscore
in Pre-Algebra/Elementary Algebra, a subscore in
Intermediate Algebra/Coordinate Geometry, and a
subscore in Plane Geometry/Trigonometry.
Tips for Taking the ACT Mathematics Test
Pace yourself.
The ACT mathematics test contains 60 questions to be
completed in 60 minutes. You have an average of 1 minute
per question. If possible, spend less time on each question
and use the remaining time allowed for this test to review
your work and return to the questions on this test that were
most difficult for you.
If you use a calculator, use it wisely.
All of the mathematics problems can be solved without
using a calculator. Many of the problems are best done
without a calculator. Use good judgment in deciding
when, and when not, to use a calculator. For example, for
some problems you may wish to do scratch work to clarify
your thoughts on the question before you begin using a
calculator to do computations.
Solve the problem.
For working out the solutions to the problems, you will
usually do scratch work in the space provided in the test
booklet. You may wish to glance over the answer choices
after reading the questions. However, working backwards
from the answer choices provided can take a lot of time
and may not be effective.
5
Locate your solution among the answer choices.
Once you have solved the problem, look for your answer
among the choices. If your answer is not included among
the choices, carefully reread the problem to see whether
you missed important information. Pay careful attention to
the question being asked. If an equation is to be selected,
check to see whether the equation you think is best can be
transformed into one of the answer choices provided.
Make sure you answer the question.
The solutions to many questions on the test will involve
several steps. Make sure your answer accounts for all the
necessary steps. Frequently, questions include answer
choices that are based on incomplete solutions.
Make sure your answer is reasonable.
Sometimes an error in computation will result in an answer
that is not practically possible for the situation described.
Always think about your answer to determine whether it is
reasonable.
Check your work.
You may arrive at an incorrect solution by making common
errors in the problem-solving process. Thus, if there is
time remaining before the end of the mathematics test, it
is important that you reread the questions and check your
answers to make sure they are correct.
Content Covered by the ACT Mathematics Test
Six content areas are included in the mathematics test:
pre-algebra, elementary algebra, intermediate algebra,
coordinate geometry, plane geometry, and trigonometry.
The questions covering pre-algebra and elementary algebra
make up the Pre-Algebra/Elementary Algebra subscore. The
questions covering intermediate algebra and coordinate
geometry make up the Intermediate Algebra/Coordinate
Geometry subscore. The questions covering plane geometry
and trigonometry make up the Plane Geometry/Trigonometry
subscore. A brief description and the approximate
percentage of the test devoted to each content area are
given below.
PRE-ALGEBRA/ELEMENTARY ALGEBRA
Pre-Algebra (20–25%). Questions in this content area are
basedon basic operations using whole numbers, decimals,
fractions, and integers; place value; square roots and
approximations; the concept of exponents; scientific notation;
factors; ratio, proportion, and percent; linear equations
in one variable; absolute value and ordering numbers by
value; elementary counting techniques and simple proba-
bility; data collection, representation, and interpretation;
andunderstanding simple descriptive statistics.
Elementary Algebra (15–20%). Questions in this content area
are based on properties of exponents and square roots,
evaluation of algebraic expressions through substitution,
using variables to express functional relationships,
understanding algebraic operations, and the solution of
quadratic equations by factoring.
INTERMEDIATE ALGEBRA/COORDINATE GEOMETRY
Intermediate Algebra (15–20%). Questions in this content
area are based on an understanding of the quadratic
formula, rational and radical expressions, absolute value
equations and inequalities, sequences and patterns, systems
of equations, quadratic inequalities, functions, modeling,
matrices, roots of polynomials, and complex numbers.
Coordinate Geometry (15–20%). Questions in this content
area are based on graphing and the relations between
equations and graphs, including points, lines, polynomials,
circles, and other curves; graphing inequalities; slope;
parallel and perpendicular lines; distance; midpoints; and
conics.
PLANE GEOMETRY/TRIGONOMETRY
Plane Geometry (20–25%). Questions in this content area
are based on the properties and relations of plane figures,
including angles and relations among perpendicular and
parallel lines; properties of circles, triangles, rectangles,
parallelograms, and trapezoids; transformations; the
concept ofproof and proof techniques; volume; and
applications of geometry to three dimensions.
Trigonometry (5–10%). Questions in this content area
are based on understanding trigonometric relations in
right triangles; values and properties of trigonometric
functions; graphing trigonometric functions; modeling using
trigonometric functions; use of trigonometric identities; and
solving trigonometric equations.
ACT Reading Test
The ACT reading test is a 40-question, 35-minute test
that measures your reading comprehension. The test
questions ask you to derive meaning from several texts by
(1) referring to what is explicitly stated and (2) reasoning to
determine implicit meanings. Specifically, questions will ask
you to use referring and reasoning skills to determine main
ideas; locate and interpret significant details; understand
sequences of events; make comparisons; comprehend
cause-effect relationships; determine the meaning of
context-dependent words, phrases, and statements; draw
generalizations; and analyze the author’s or narrator’s
voice and method. The test comprises four sections, each
containing one long or two shorter prose passages that
are representative of the level and kinds of text commonly
encountered in first-year college curricula. Each passage is
preceded by a heading that identifies what type of passage
it is (for example, “Literary Narrative”), names the author,
and may include a brief note that helps in understanding
the passage. Each section contains a set of multiple-choice
test questions. These questions do not test the rote recall
of facts from outside the passage, isolated vocabulary
items, or rules of formal logic. In sections that contain two
short passages, some of the questions involve both of the
passages in the section.
6
Three scores are reported for the ACT reading test: a total
test score based on all 40 questions, a subscore in Social
Studies/Sciences reading skills (based on the 20 questions
on the social studies and natural sciences passages), and
a subscore in Arts/Literature reading skills (based on the 20
questions on the literary narrative and humanities passages).
Tips for Taking the ACT Reading Test
Pace yourself.
The ACT reading test contains 40 questions to be
completed in 35 minutes. If you spend 2–3 minutes reading
the passage(s) in each section, then you will have about
35 seconds to answer each question. If possible, spend
less time on the passages and the questions and use the
remaining time allowed for this test to review your work and
return to the questions on this test that were most difficult
for you.
Read each passage carefully.
Before you begin answering a question, read the entire
passage (or two short passages) carefully. Be conscious
of relationships between or among ideas. You may
make notes in the test booklet about important ideas in the
passages.
Refer to the passages when answering the questions.
Answers to some of the questions will be found by referring
to what is explicitly stated in the text. Other questions will
require you to determine implicit meanings and to draw
conclusions, comparisons, and generalizations. Consider
the text before you answer any question.
Content Covered by the ACT Reading Test
The reading test is based on four types of reading
selections: the social studies, the natural sciences,
literary narrative, and the humanities. A subscore in Social
Studies/Sciences reading skills is based on the questions
on the social studies and the natural sciences passages,
and a subscore in Arts/Literature reading skills is based
on the questions on the literary narrative and humanities
passages. A brief description and the approximate
percentage of the test devoted to each type of reading
selection are given below.
Social Studies (25%). Questions in this category are
based on passages in the content areas of anthropology,
archaeology, biography, business, economics, education,
geography, history, political science, psychology, and
sociology.
Natural Sciences (25%). Questions in this category are
based on passages in the content areas of anatomy,
astronomy, biology, botany, chemistry, ecology, geology,
medicine, meteorology, microbiology, natural history,
physiology, physics, technology, and zoology.
Literary Narrative (25%) or Prose Fiction (25%). Questions
in the Literary Narrative category are based on passages
from short stories, novels, memoirs, and personal essays.
Questions in the Prose Fiction category are based on
passages from short stories and novels.
Humanities (25%). Questions in this category are based on
passages in the content areas of architecture, art, dance,
ethics, film, language, literary criticism, music, philosophy,
radio, television, and theater. Questions may be based on
passages from memoirs and personal essays.
ACT Science Test
The ACT science test is a 40-question, 35-minute test that
measures the interpretation, analysis, evaluation, reasoning,
and problem-solving skills required in the natural sciences.
The test presents several sets of scientific information, each
followed by a number of multiple-choice test questions. The
scientific information is conveyed in one of three different
formats: data representation (graphs, tables, and other
schematic forms), research summaries (descriptions of
several related experiments), or conflicting viewpoints
(expressions of several related hypotheses or views that
are inconsistent with one another). The questions require
you to recognize and understand the basic features of, and
concepts related to, the provided information; to examine
critically the relationship between the information provided
and the conclusions drawn or hypotheses developed;
and to generalize from given information to gain new
information, draw conclusions, or make predictions.
You are not permitted to use a calculator on the ACT
science test.
One score is reported for the ACT science test: a total test
score based on all 40 questions.
Tips for Taking the ACT Science Test
Pace yourself.
The ACT science test contains 40 questions to be completed
in 35 minutes. If you spend about 2 minutes reading each
passage, then you will have about 30 seconds to answer
each question. If possible, spend less time on the passages
and the questions and use the remaining time allowed for
this test to review your work and return to the questions on
this test that were most difficult for you.
Read the passage carefully.
Before you begin answering a question, read the scientific
material provided. It is important that you read the entire
text and examine any tables, graphs, or figures. You may
want to make notes about important ideas in the information
provided in the test booklet. Some of the information
sets will describe experiments. You should consider the
experimental design, including the controls and variables,
because questions are likely to address this component of
scientific research.
Note different viewpoints in passages.
Some material will present conflicting points of view,
and the questions will ask you to distinguish among the
various viewpoints. It may be helpful for you to make notes
summarizing each viewpoint next to that section in the test
booklet.
Content Covered by the ACT Science Test
The content of the science test includes biology,
chemistry, physics, and the Earth/space sciences
(for example, geology, astronomy, and meteorology).
Advanced knowledge in these subjects is not required,
but knowledge acquired in general, introductory science
courses is needed to answer some of the questions. The
test emphasizes scientific reasoning skills over recall of
scientific content, skill in mathematics, or reading ability.
The scientific information is conveyed in one of three
different formats.
7
Data Representation (30–40%). This format presents
graphic and tabular material similar to that found in science
journals and texts. The questions associated with this
format measure skills such as graph reading, interpretation
of scatter plots, and interpretation of information presented
in tables.
Research Summaries (45–55%). This format provides
descriptions of one or more related experiments. The
questions focus upon the design of experiments and the
interpretation of experimental results.
Conflicting Viewpoints (15–20%). This format presents
expressions of several hypotheses or views that, being
based on differing premises or on incomplete data, are
inconsistent with one another. The questions focus upon
the understanding, analysis, and comparison of alternative
viewpoints or hypotheses.
ACT Writing Test (Optional)
If you register for the ACT with writing, you will take the ACT
writing test (which must be completed in English) after you
complete the four multiple-choice tests.
The ACT writing test is a 40-minute essay test that
measures your writing skills—specifically those writing
skills emphasized in high school English classes and in
entry-level college composition courses. The test describes
an issue and provides three different perspectives on
the issue. You are asked to “evaluate and analyze” the
perspectives; to “state and develop” your own perspective;
and to “explain the relationship” between your perspective
and those given. Your score will not be affected by the
perspective you take on the issue.
Taking the writing test will not affect your scores on the
multiple-choice tests or your Composite score. Rather, you
will receive a single subject-level writing score on a scale
of 1–36 and five additional scores: an English Language
Arts score on a scale of 1–36 and scores for four domains
of writing competencies (Ideas and Analysis, Development
and Support, Organization, and Language Use and
Conventions) on a scale of 2–12.
Two trained readers will score your essay from 1-6 in each
of four writing domains. Each domain score represents
the sum of the two readers’ scores. Your Writing Score is
calculated from your domain scores and is reported on a
scale of 1–36. Your domain scores do not necessarily add
up to your reported Writing Score.
Tips for Taking the ACT Writing Test
Pace yourself.
The ACT writing test contains one question to be
completed in 40 minutes. When asked to write a timed
essay, most writers find it useful to do some planning
before they write the essay, and to do a final check of the
essay when it is finished. It is unlikely that you will have time
to draft, revise, and recopy your essay.
Plan.
Before writing, carefully read and consider all prompt
material. Be sure you understand the issue, its
perspectives, and your essay task. The prewriting
questions included with the prompt will help you analyze
the perspectives and develop your own. Use these
questions to think critically about the prompt and generate
effective ideas in response. Ask yourself how your ideas
and analysis can best be supported and organized in a
written argument. Use the prewriting space in your test
booklet to structure or outline your response.
Write.
Establish the focus of your essay by making clear your
argument and its main ideas. Explain and illustrate your
ideas with sound reasoning and meaningful examples.
Discuss the significance of your ideas: what are the
implications of what you have to say, and why is your
argument important to consider? As you write, ask yourself
if your logic is clear, you have supported your claims, and
you have chosen precise words to communicate your
ideas.
Review your essay.
Take a few minutes before time is called to read over your
essay. Correct any mistakes. If you find any words that are
hard to read, recopy them. Make corrections and revisions
neatly, between the lines. Do not write in the margins. Your
readers know you had only 40 minutes to compose and
write your essay. Within that time limit, try to make your
essay as polished as you can.
Practice.
There are many ways to prepare for the ACT writing
test. These include reading newspapers and magazines,
listening to news analyses on television or radio, and
participating in discussions and debates.
One of the best ways to prepare for the ACT writing test
is to practice writing with different purposes for different
audiences. The writing you do in your classes will help you.
So will writing essays, stories, editorials, a personal journal,
or other writing you do on your own.
It is also a good idea to practice writing within a time limit.
Taking the practice ACT writing test will give you a sense
of how much additional practice you may need. You might
want to take the practice ACT writing test even if you do
not plan to take the ACT with writing, because this will help
build skills that are important in college-level learning and
in the world of work.
8
3
What to Expect
on Test Day
Reporting Time
For National and International test dates, you must report
to the test center by the time stated on your ticket, normally
8:00 a.m. If you are late, you will not be admitted to test. If
your ticket does not list a specific room, test center staff or
posted signs will direct you.
Requirements for Admission
At check-in, you will be required to show both your
paper ticket and acceptable photo ID or you will not be
admitted to test. See ID requirements on your ticket or
at www.actstudent.org.
In the Test Room
The supervisor or proctor will direct you to a seat. If
you need a left-handed desk, tell your supervisor as
you enter.
Do not leave the test room after you have been admitted.
Only pencils, erasers, a permitted calculator, and your
ticket will be allowed on your desk.
You will be required to put all other personal belongings
away.
You may not use tobacco in any form or have food or
drink (including water) in the test room. You may have
snacks and drinks outside the test room during break.
Testing will begin as soon as all examinees present at
8:00 a.m. are checked in and seated.
Listen carefully to all directions read by your supervisor.
It is important that you follow all directions carefully.
On some test dates, ACT tries out questions to develop
future versions of the tests. You may be asked to take
a fifth test, the results of which will not be reflected in
your reported scores. The fifth test could be multiple-
choice or one for which you will create your own
answers. Please try your best on these questions,
because your participation can help shape the future of
the ACT. If you are in a test room where the fifth test is
administered, you will be dismissed at about 12:35 p.m.
Prohibited Behavior at the Test
Center
The following behaviors are prohibited. You will be
dismissed and your answer document will not be scored if
you are found:
Filling in or altering responses on a test section on your
answer sheet or continuing to complete the essay after
time has been called on that test section. This means
that you cannot make any changes to a test section
outside of the designated time for that section, even to
fix a stray mark or accidental keystroke.
Looking back at a test section on which time has
already been called.
Looking ahead in the test booklet.
Looking at another person’s test booklet or answer
document.
Giving or receiving assistance by any means.
Discussing or sharing of test content, test form
identification numbers, or answers during test
administration, during breaks, or after the test is
prohibited.
Using a prohibited calculator (www.actstudent.org).
Using a calculator on any test section other than
Mathematics.
Sharing a calculator with another person.
Using a watch with recording, internet, or
communication capabilities.
Using any electronic device at any time during testing
or during break other than an approved calculator
or watch. All other electronic devices, including cell
phones and wearable devices, must be turned off and
placed out of reach from the time you are admitted to
test until you are dismissed after testing concludes.
Attempting to memorize test-related information or
otherwise remove test materials, including questions or
answers, from the test room in any way.
Using highlight pens, colored pens or pencils, notes,
dictionaries, or other aids.
Using scratch paper (unless an exception applies).
o Specific instructions will be provided on test day if
ACT authorizes you to use scratch paper, including
the section(s) on which ACT has authorized its use.
o If you are permitted to use scratch paper, you may
only use paper that ACT has authorized and/or
provided to you.
Not following instructions or abiding by the rules of the
test center.
Exhibiting confrontational, threatening, or unruly
behavior; or violating any laws. If ACT suspects you are
engaging in criminal activities, such activities will be
reported to law enforcement agencies.
Allowing an alarm to sound in the test room or creating
any other disturbance.
All items brought into the test center, such as hats, purses,
backpacks, cell phones, calculators, and other electronic
devices may be searched at the discretion of ACT and its
testing staff. ACT and its testing staff may confiscate and
retain for a reasonable period of time any item suspected
of having been used, or being capable of being used, in
violation of this list of prohibited behaviors. ACT may also
provide such items to third parties in connection with an
investigation conducted by ACT or others. ACT and its
testing staff shall not be responsible for lost, stolen, or
damaged items.
9
Voiding Your Answer
Documents on Test Day
If you have to leave the test center before completing all
your tests, you must decide whether or not you want your
answer document scored and inform your supervisor of
your decision. If you do not, your answer document will
be scored.
Once you break the seal on your multiple-choice test
booklet, you cannot request a Test Date Change. If you
do not complete all your tests and want to test again, you
will have to pay the full fee for your test option again. If
you want to take the ACT again, see www.actstudent.org
for your options. Once you begin filling out your answer
document, you cannot change from one test option to
another.
Testing More Than Once
You may not receive scores from more than one test taken
during a scheduled National or International test date. For
example, you may test on Saturday or on an authorized
non-Saturday date or on a rescheduled test date—but
not on more than one of those days. If you are admitted
and allowed to test a second time, we will report only the
scores from the first test. The second set of scores will be
cancelled without refund.
Test Information Release
On certain National test dates, if you test at a National
test center, you may order (for an additional fee) a copy
of the test questions, a copy of your answers, a list of
correct answers, and scoring instructions. This service is
not available for all test dates or for other testing programs
(e.g., International, State and District, Special). If you want to
request and pay for this service, check www.actstudent.org
to see which test dates offer this service.
4
Taking the Practice Tests
Take the practice tests under conditions as similar as
possible to those you will experience on test day. The
following tips will help you:
The four multiple-choice tests require 2 hours and
55minutes. Take them in order in one sitting, with a
10- to 15-minute break between Tests 2 and 3.
You will need only sharpened No. 2 pencils with good
erasers. Remove all other items from your desk. You will
not be allowed to use scratch paper.
If you plan to use a permitted calculator on the
mathematics test, use the same one you will use on test
day.
Use a digital timer or clock to time yourself on each
practice test. Set your timer for five minutes less than
the time allowed for each test so you can get used to
the verbal announcement of five minutes remaining.
Give yourself only the time allowed for each test.
Detach and use the sample multiple-choice answer
document on pages 63–64.
Read the test directions on the first page of the practice
multiple-choice tests. These are the same directions
that will appear on your test booklet on test day.
Start your timer and begin with Test 1. Continue through
Test 4, taking a 10- to 15-minute break between Tests
2 and 3. If you do not plan to take the ACT with writing,
score your multiple-choice tests using the information
beginning on page 56.
If you plan to take the ACT with writing, read the
directions on the first page of the practice ACT writing
test (page 53). These are the same directions that will
appear on your test booklet on test day. Start your
timer, then read the prompt on page 54. After you
understand what the prompt is asking you to do, plan
your essay and then write it on lined paper. (On test
day, your answer document will have lined pages for
you to write your essay.) Score your essay using the
information on pages 61–62.
10
11
Practice Multiple-Choice Tests
Form 1572CPRE
This booklet contains tests in English, mathematics,
reading, and science. These tests measure skills and
abilities highly related to high school course work and
success in college. Calculators may be used on the
mathematics test only.
The questions in each test are numbered, and the
suggested answers for each question are lettered. On the
answer document, the rows of ovals are numbered to
match the questions, and the ovals in each row are lettered
to correspond to the suggested answers.
For each question, first decide which answer is best.
Next, locate on the answer document the row of ovals
numbered the same as the question. Then, locate the oval
in that row lettered the same as your answer. Finally, fill in
the oval completely. Use a soft lead pencil and make your
marks heavy and black. Do not use ink or a mechanical
pencil.
Mark only one answer to each question. If you change
your mind about an answer, erase your first mark thoroughly
before marking your new answer. For each question, make
certain that you mark in the row of ovals with the same
number as the question.
Only responses marked on your answer document will
be scored. Your score on each test will be based only on
the number of questions you answer correctly during the
time allowed for that test. You will not be penalized for
guessing. It is to your advantage to answer every
question even if you must guess.
You may work on each test only when the testing staff
tells you to do so. If you finish a test before time is called
for that test, you should use the time remaining to
reconsider questions you are uncertain about in that test.
You may not look back to a test on which time has already
been called, and you may not go ahead to another test. To
do so will disqualify you from the examination.
Lay your pencil down immediately when time is called at
the end of each test. You may not for any reason fill in or
alter ovals for a test after time is called for that test. To do
so will disqualify you from the examination.
Do not fold or tear the pages of your test booklet.
DO NOT OPEN THIS BOOKLET
UNTIL TOLD TO DO SO.
Directions
© 2015 by ACT, Inc. All rights reserved.
NOTE: This test material is the confidential copyrighted property of
ACT, Inc., and may not be copied, reproduced, sold, or otherwise
transferred without the prior express written permission of ACT, Inc.
Violators of ACT’s copyrights are subject to civil and criminal penalties.
PO BOX 168
I
OWA CITY, IA 52243-0168
EXAMINEE STATEMENT, CERTIFICATION, AND SIGNATURE
1. Read the following Statement: By opening this test booklet, I agree to comply with and be bound by the Terms and
Conditions: Testing Rules and Policies for the ACT
®
provided in the ACT registration materials for this assessment,
including those concerning test security, score cancellation, examinee remedies, arbitration, and consent to the
processing of
my personally identifying information, including the collection, use, transfer and disclosure of
information as described in the ACT Privacy Policy (available at www.act.org/privacy.html).
International Examinees: By my signature I am also providing my consent to ACT to transfer my personally
identifying information to the United States to ACT, or a third party service provider for processing, where it will be
subject to use and disclosure under the laws of the United States. I acknowledge and agree that it may also be
accessible to law enforcement and national security authorities in the United States.
I understand that ACT owns the assessment questions and responses and affirm that I will not share any
assessment questions or responses with anyone by any form of communication before, during
, or after the
assessment administration. I understand that assuming anyone else’s identity to take this assessment is strictly
prohibited and may violate the law and subject me to legal penalties.
2. Copy the Certification shown below (only the text in italics) on the lines provided. Write in your normal handwriting.
Certification: I agree to the Statement above and certify that I am the person whose nam
e appears on this form.
3. Sign your name as you would any official document and enter today’s date.
Your Signature Today’s Date
PASSAGE I
The Triangular Snowflake
[1]
Snowflakes form from tiny water droplets, following
a specific process of chemical bonding as they freeze,
which results in a six-sided figure. The raretriangular
snowflake, similarly, confounded scientists for years
because it apparently defied the basic laws of chemistry.
[A] The seemingly triangular shape of those snowflakes
suggests that forming through a different process of
chemical bonding. [B] By re-creating snowflake formation,
a discovery has revealed to scientists Kenneth Libbrecht
and Hannah Arnold the cause of this apparent variation.
[2]
Snowflakes begin to form when water in the
atmosphere freezes it causes the water molecules
to bond into a hexagonal shape. During the flake’s
descent from Earth’s upper atmosphere, other water
vapor molecules bumps
into the hexagonal structure.
1. A. NO CHANGE
B. form, from tiny, water droplets,
C. form from tiny, water, droplets
D. form, from tiny water droplets
2. F. NO CHANGE
G. for example,
H. additionally,
J. however,
3. A. NO CHANGE
B. the manner in which formation
C. which had formed
D. that they form
4. F. NO CHANGE
G. the discovery of the cause of this apparent variation
has been made by scientists Kenneth Libbrecht and
Hannah Arnold.
H. scientists Kenneth Libbrecht and Hannah Arnold
have discovered the cause of this apparent variation.
J. the cause of this apparent variation has been dis-
covered by scientists Kenneth Libbrecht and
Hannah Arnold.
5. A. NO CHANGE
B. freezes, causing
C. freezes, it causes
D. freezes, this causes
6. F. NO CHANGE
G. has bumped
H. bumped
J. bump
ENGLISH TEST
45 Minutes—75 Questions
DIRECTIONS: In the five passages that follow, certain
words and phrases are underlined and numbered. In
the right-hand column, you will find alternatives for the
underlined part. In most cases, you are to choose the
one that best expresses the idea, makes the statement
appropriate for standard written English, or is worded
most consistently with the style and tone of the passage
as a whole. If you think the original version is best,
choose “NO CHANGE.” In some cases, you will find in
the right-hand column a question about the underlined
part. You are to choose the best answer to the question.
You will also find questions about a section of the pas-
sage, or about the passage as a whole. These questions
do not refer to an underlined portion of the passage, but
rather are identified by a number or numbers in a box.
For each question, choose the alternative you consider
best and ll in the corresponding oval on your answer
document. Read each passage through once before you
begin to answer the questions that accompany it. For
many of the questions, you must read several sentences
beyond the question to determine the answer. Be sure
that you have read far enough ahead each time you
choose an alternative.
GO ON TO THE NEXT PAGE.
ACT-1572CPRE
11
1
2
3
4
4
5
6
12
13
Bypassing the liquid water phase,
those molecules
condense directly onto the established hexagonal pattern.
As a result, the flake grows outward into bigger and more
complex hexagonal arrangements surrounding the original
hexagonal shape at the center of the flake. [C]
[3]
In 2009, Libbrecht and Arnold’s experiments
revealed that triangular snowflakes begin with the
same process of chemical bonding and forms a hexagonal
shape. The triangular shape is an illusion resulting from
one significant addition to the process dust.
[4]
Triangular snowflakes begin to form when a tiny
dust particle or other such impurity collides with the
flake as it falls, thereby pushing one edge upward. [D]
The downward edge of the snowflake encounters more
wind resistance than the rest of the flake. The greater
the pressure from the wind, causes bonds to form
quick at this edge than in the rest of the snowflake.
[5]
The resulting snowflake has three long sides and
three sides that are so short they are difficult to detect.
Although these snowflakes appear to have a triangular
shape—they actually have a hexagonal pattern. Such
snowflakes offer evidence that even when impurities
interfere, the basic laws of chemistry still apply.
7. If the writer were to delete the underlined portion
(adjusting the capitalization as needed), the sentence
would primarily lose:
A. an explanation of the process water molecules
undergo to change from liquid to vapor to solid.
B. a detail that mentions a step some water molecules
skip in changing from vapor to solid.
C. a visual description of what water vapor molecules
look like.
D. an explanation of how molecules react to various
air temperatures.
8. F. NO CHANGE
G. were they to form
H. if they formed
J. form
9. A. NO CHANGE
B. process is
C. process:
D. process;
10. F. NO CHANGE
G. pressure from the wind, which
H. the pressure, as the wind
J. pressure from the wind
11. A. NO CHANGE
B. more quickly
C. most quickly
D. quickest
12. F. NO CHANGE
G. shape,
H. shape;
J. shape:
13. Which choice most effectively concludes the sentence
and the essay?
A. NO CHANGE
B. scientists can be certain that a solution to even the
most confusing event will be found.
C. snowflakes will still fall if atmospheric conditions
are favorable.
D. snowflakes come in many different shapes and
sizes.
GO ON TO THE NEXT PAGE.
ACT-1572CPRE
11
7
8
9
10
11
12
13
14. The writer is considering adding the following sen-
tence to the essay:
This growth can take the form of either
branching (which forms stable, symmetrical
shapes) or faceting (which forms unstable,
complex shapes).
If the writer were to add this sentence, it would most
logically be placed at Point:
F. A in Paragraph 1.
G. B in Paragraph 1.
H. C in Paragraph 2.
J. D in Paragraph 4.
PASSAGE II
Climbing Mt. Fuji
[1]
Bundled up in wool sweaters and thick
coats, and we watched the sun setting on Mt. Fuji
in Japan. It was August and our clothes were stifling,
but we would have needed the warmth from our bodies
sealed around us as we hiked into the high altitudes.
Three friends and I stepped away from the crowd of
other hikers and spoke our intention: Sunset at the
base, sunrise at the top. [A]
[2]
As we hiked, a patchwork of clouds swept across
the darkening sky, hiding all traces of our surroundings
outside our flashlights beams. The trail gradually changed
from compact dirt to a jumble of volcanic rocks. [B]
15. Suppose the writer’s primary purpose had been to offer
an example of a discovery that changed the way scien-
tists viewed the basic laws of chemistry. Would this
essay accomplish that purpose?
A. Yes, because it describes how the observation of
triangular snowflakes has led scientists to discover
that their understanding of the basic laws of chem-
istry is flawed.
B. Yes, because it describes how scientists have
applied the knowledge they’ve gained through
studying snowflakes to other areas of chemistry.
C. No, because it focuses on how scientists are strug-
gling to determine how triangular snowflakes are
formed.
D. No, because it explains that triangular snowflakes
appeared to, but don’t actually, violate the basic
laws of chemistry.
16. F. NO CHANGE
G. coats while watching
H. coats, we watched
J. coats watching
17. A. NO CHANGE
B. would need
C. will need
D. need
GO ON TO THE NEXT PAGE.
ACT-1572CPRE
11
Questions 14 and 15 ask about the preceding passage as a whole.
16
17
14
15
We tried to steady ourselves with our
walking sticks but slipped and stumbled
because of the jumbled rocks we were slipping on.
[3]
Every thousand feet, we came to a small station
constructed of tin and cement, barely able to block
the wind. At each one, we noted the roof piled high on
fallen rocks and felt both unsettled and reassured by this
evidence of the station’s protective ability. We rested
uneasily for a moment as a clerk burned the station brand
into our walking sticks which it was proof of our progress
through the darkness.
[4]
As we neared the summit, the whole group of
hikersthinly spread across the mountain for most of
the routecondensed, forming an illuminated line along
the trail. [C] Our pace slowed. Progressing along the trail,
we reached the summit just five minutes before dawn. [D]
In the half-light of the rising sun: we began to make
out the dark lines of the cliffsat the crater’s edge.
18. F. NO CHANGE
G. even though we used our walking sticks.
H. despite any efforts to remain steady.
J. with each step.
19. A. NO CHANGE
B. piling high with
C. piled high with
D. piling high on
20. F. NO CHANGE
G. sticks, it was proof of
H. sticks, proof of
J. sticks proved
21. A. NO CHANGE
B. the most part
C. majority
D. more
22. F. NO CHANGE
G. they formed
H. there was
J. we saw
23. Which choice emphasizes the slowness of the ascent
and supports the idea that the narrator’s group of
friends did not set their own pace?
A. NO CHANGE
B. Able to advance only a few steps at a time,
C. Moving forward with each step,
D. Climbing higher in altitude,
24. F. NO CHANGE
G. sun
H. sun,
J. sun;
25. A. NO CHANGE
B. cliffs at the craters
C. cliffs at the crater’s
D. cliffs at the craters
GO ON TO THE NEXT PAGE.
ACT-1572CPRE
11
18
19
20
21
22
23
24
25
26. If the writer were to delete the preceding sentence, the
paragraph would primarily lose:
F. a restatement of an idea that emphasizes the
hikers anticipation when they reached the summit.
G. a statement that introduces the idea of waiting,
which is the focus of the following paragraph.
H. an unnecessary detail that contradicts information
presented earlier in the paragraph.
J. a clear image that conveys what the hikers saw
when they reached the summit.
27. A. NO CHANGE
B. Furthermore,
C. Once again,
D. Finally,
28. Which choice most dramatically emphasizes the
ruggedness of the landscape?
F. NO CHANGE
G. shattered over
H. smothered
J. went over
30. Suppose the writer’s primary purpose had been to
describe the experience of doing something difficult.
Would this essay accomplish that purpose?
F. Yes, because it tells about a variety of challenges
the hikers faced along their journey.
G. Yes, because it focuses primarily on the hikers
need for walking sticks and other tools to make it
up the trail.
H. No, because it focuses on the rewarding nature of
the experience but does not describe the hike as
challenging.
J. No, because it focuses mainly on the beauty of the
surrounding landscape.
31. A. NO CHANGE
B. boy named Juan Quezada
C. boy, named Juan Quezada
D. boy named Juan Quezada,
GO ON TO THE NEXT PAGE.
ACT-1572CPRE
11
We crouched down on jutting pieces of rock and waited for
the shifting clouds to clear. We waited for the sun. :
[5]
Generally, a sudden gap in the clouds left us blinking
as the sunlight squelched out the severe landscape of
gray volcanic rock. We leaned against each other, spent.
Perhaps there is truth in the old Japanese saying: A wise
man climbs Mt. Fuji, but only a fool climbs it twice.
29. The writer wants to add the following sentence to the
essay:
We clipped small flashlights onto our coats,
picked up our walking sticks, and started up
the trail with the other hikers as the sun
dipped below the trees.
The sentence would most logically be placed at Point:
A. A in Paragraph 1.
B. B in Paragraph 2.
C. C in Paragraph 4.
D. D in Paragraph 4.
PASSAGE III
The Pottery of Mata Ortiz
In the early 1950s, a twelve-year-old
boy named, Juan Quezada, gathered firewood
in the mountains near the village of Mata Ortiz
in Chihuahua, Mexico. Though he dreamed of
becoming an artist, Quezada spent all of his free
time selling firewood to help support his family.
Questions 29 and 30 ask about the preceding passage as a whole.
27
28
31
16
17
In the mountains, Quezada found shards of
pots, and an occasional complete pot,
painted with
intricate red and black designs. These were artifacts
from his ancestors, the Paquimé (or Casas Grandes)
Indians, who lived in the area from about AD 1000
to AD 1400. Fascinated by the geometric designs,
Quezada wondered, if he could make pots like these?
B He dug the clay, soaked it, and tried to shape it
into a pot. In time, he figured out how his ancestors had
mixed the clay with volcanic ash to keep it from cracking
and had used minerals found nearby to create paints. When
it was time to paint his pots, Quezada designed his own
complex geometric patterns.
As an adult, Quezada found a job with the
railroad, but he always made time for his art. By 1976
he was selling pots to travelers and had taught several
members of his family how to make pots. Three of
Quezada’s pots were discovered in a junk shop in
New Mexico by anthropologist Spencer MacCallum,
who at first thought they were prehistoric. D
His search for their creator led him to Mata
Ortiz and an eventual partnership with Quezada.
32. Which of the following alternatives to the underlined
portion would NOT be acceptable?
F. potsalong with an occasional complete pot
G. pots, along with an occasional complete pot,
H. pots, (and an occasional complete pot)
J. pots (and an occasional complete pot)
33. A. NO CHANGE
B. wondered if he could make pots like these.
C. wondered, if he could make pots like these.
D. wondered if he could make pots like these?
34. Which of the following true statements would provide
the best transition from the preceding paragraph to this
paragraph?
F. The village of Mata Ortiz is only three streets wide
but stretches for a mile between the Casas Grandes
River and the railroad tracks.
G. The patterns on Mata Ortiz pottery that Quezada
admired are based on the techniques of the ancient
Paqui.
H. Quezada began working with clay from the
mountains.
J. Quezada’s painted designs became increasingly
complex.
35. A. NO CHANGE
B. a dedication to teaching
C. a teacher of
D. has taught
36. In the preceding sentence, the clause who at first
thought they were prehistoric primarily serves to
indicate:
F. how closely Quezada had created his pots within
the Paquimé tradition.
G. that Quezada’s technique as a potter wasnt very
well developed yet.
H. how strikingly simple Quezada’s pots were in
shape and design.
J. that the style of Quezada’s pots was outmoded.
37. A. NO CHANGE
B. lead himself
C. led himself
D. lead him
38. Which choice most strongly suggests that Quezada’s
partnership with MacCallum was not formed right
away upon MacCallum’s arrival in Mata Ortiz?
F. NO CHANGE
G. a circumstantial
H. a momentary
J. a timely
GO ON TO THE NEXT PAGE.
ACT-1572CPRE
11
32
33
35
37
38
MacCallum showed Quezada’s pots to art dealers in the
United States, the places in which art galleries were soon
offering Quezada thousands of dollars for them.
[1] Quezada helped his village with the money he
earned selling pottery, but he wanted to do more so. [2] So
he taught people from Mata Ortiz to make pots. [3] Today
there are more than four hundred potters around, all of
which make their pots by hand, following the traditions
of the Paquimé Indians. [4] The village is thriving, and
many museums proudly display the pottery of Mata Ortiz.
[5] Each artist brought something unique to they’re
creations. L
39. A. NO CHANGE
B. and it would happen there that
C. where
D. DELETE the underlined portion.
40. F. NO CHANGE
G. more then that.
H. more of them.
J. more.
41. A. NO CHANGE
B. people creating art now,
C. potters in Mata Ortiz,
D. DELETE the underlined portion and place a
comma after the word hundred.
42. F. NO CHANGE
G. whom
H. them
J. who
43. A. NO CHANGE
B. his or herselves
C. hers or his
D. his or her
44. For the sake of the logic and coherence of this para-
graph, Sentence 5 should be placed:
F. where it is now.
G. before Sentence 1.
H. after Sentence 1.
J. after Sentence 2.
45. Suppose the writer’s primary purpose had been to write
an essay summarizing the history of pottery making in
Mexico. Would this essay accomplish that purpose?
A. Yes, because it discusses ancient pottery shards
and complete pots from the Paquimé Indians and
compares that pottery to modern designs.
B. Yes, because it demonstrates the quality of the
ancient pottery of the Mata Ortiz area.
C. No, because it focuses instead on how one artist
based his creations on ancient pottery techniques
and shared those techniques with other artists.
D. No, because it focuses instead on describing the
Casas Grandes culture in ancient Mexico.
GO ON TO THE NEXT PAGE.
ACT-1572CPRE
11
Question 45 asks about the preceding passage
as a whole.
39
40
41
42
43
18
19
46. F. NO CHANGE
G. they’re
H. their
J. its
47. A. NO CHANGE
B. alike, regularly filling
C. alike, regularly fill
D. alike regularly fill
48. F. NO CHANGE
G. consequently,
H. however,
J. in fact,
49. A. NO CHANGE
B. there to
C. whom
D. they
50. F. NO CHANGE
G. frieze; into which are carved
H. frieze. Into which are carved
J. frieze, carved into it are
51. The writer is considering adding the following
sentence:
Masks figured prominently in classical Greek
theater performances, in part due to the fact
that one actor would usually play several
characters.
Should the writer make this addition here?
A. Yes, because it connects the paragraph’s point
about theatrical masks to the larger subject of clas-
sical Greek theater.
B. Yes, because it explains the masks significance to
classical Greek theater and architecture.
C. No, because it only addresses classical Greek
theater and doesnt include information about
Roman theater.
D. No, because it deviates from the paragraphs focus
on the Lyceum Theatres architecture.
GO ON TO THE NEXT PAGE.
ACT-1572CPRE
11
PASSAGE IV
Beaux Arts Architecture in the Spotlight
On West 45th Street in New York City, wedged
between buildings more than twice it’s height, stands
the Lyceum Theatre. Tourists and New Yorkers
alike regularly filling this theater to its 900-seat
capacity. Most are there to attend a performance;
a few, for example, are likely to be architecture buffs
they come to admire the stunning building itself. Built in
1903, the theater exemplifies the Beaux Arts architectural
style, which fuses elements of classical Greek and Roman
design with Renaissance and Baroque details.
The Beaux Arts revival of classical Greek and Roman
architecture is apparent on first view of the theater. The
Lyceum’s facadethe exterior front, or face, of the
buildingfeatures half a dozen Corinthian columns.
Above the columns extends a horizontal stone band
called a frieze; carved into it are the classical theatrical
masks that represent comedy and tragedy. S
46
47
48
49
50
Demonstrating the Beaux Arts infusion of
Renaissance and Baroque details, tall, arched French
windows, symmetrically placed between the columns,
lighten the imposing gray limestone structure. [A]
Above the windows and frieze, an exterior balcony spans
the width of the gray building. [B] The balcony is fenced
with a balustrade, a stone railing supported by a row
of waist-high, vase-shaped pillars. [C] The ornate
interior of the building is consistent with its elaborate
exterior. [D] Not just one but two marble-finished
grand staircases lead from the foyer to the midlevel
seating area, called the mezzanine. Inside the theater
itself, elegant chandeliers illuminate rose-colored walls
that have gold accents. In keeping with sumptuous
Beaux Arts style, curved rows of plush purple chairs
embrace the stage. XY
52. F. NO CHANGE
G. gray limestone
H. limestone
J. DELETE the underlined portion.
53. A. NO CHANGE
B. balustrade. Which is
C. balustrade. It being
D. balustrade, this is
54. F. NO CHANGE
G. elegantly chandelier illuminates
H. elegantly chandelier illuminate
J. elegant chandeliers illuminates
55. Which choice maintains the essay’s positive tone and
most strongly mimics the elaborate style of decor
being described at this point in the essay?
A. NO CHANGE
B. embellished with myriad gold accents.
C. marred with gaudy accents of gold.
D. accented with gold.
56. If the writer were to delete the preceding sentence, the
essay would primarily lose details that:
F. illustrate one of the Lyceum Theatre’s features that
deviates from Beaux Arts architecture.
G. contribute to the description of the Lyceum
Theatre’s elaborate interior.
H. support the essay’s claim that Beaux Arts architec-
ture was most popular in the twentieth century.
J. clarify an unfamiliar architectural term used in the
essay.
57. The writer wants to divide this paragraph into two in
order to separate details about the building’s outdoor
features from details about its indoor features. The best
place to begin the new paragraph would be at Point:
A. A.
B. B.
C. C.
D. D.
GO ON TO THE NEXT PAGE.
ACT-1572CPRE
11
52
53
54
55
20
21
GO ON TO THE NEXT PAGE.
ACT-1572CPRE
11
58. F. NO CHANGE
G. In the same manner, patrons
H. On one hand, patrons
J. For instance, patrons
59. A. NO CHANGE
B. adding enhancement to the experience of
C. adding to the experience of
D. enhancing
60. Suppose the writer’s primary purpose had been to
explain how a building illustrates a particular architec-
tural style. Would this essay accomplish that purpose?
F. Yes, because it describes the architectural styles of
several New York theater buildings.
G. Yes, because it enumerates a number of the
Lyceum Theatre’s Beaux Arts features.
H. No, because it focuses more specifically on the set
design for the Lyceum Theatre’s productions.
J. No, because it focuses on more than one architec-
tural style.
61. A. NO CHANGE
B. factually inaccuracies.
C. factual inaccuracies.
D. factually inaccurate.
62. F. NO CHANGE
G. birth: she falsely lists
H. birth; falsely listing
J. birth, falsely listing:
63. A. NO CHANGE
B. has mattered
C. had mattered
D. matter
64. F. NO CHANGE
G. little. For
H. little; for
J. little,
65. Given that all the choices are true, which one provides
the best transition into the rest of the essay?
A. NO CHANGE
B. Born in Cork, Ireland, in 1837, Jones immigrated
to the United States in the mid-1800s.
C. Rather, it’s the story of her public persona, the rad-
ical labor activist Mother Jones.”
D. Instead, this essay will show you why Joness role
in history is so important.
Question 60 asks about the preceding passage
as a whole.
Patrons credit the handsome Beaux Arts aesthetic
with adding enhancement to their theatergoing experience.
Though smaller and more cramped than many newer
theatersaudience members often note that legroom is
limitedthe Lyceum’s distinctive atmosphere continues
to delight theater fans as well as architecture enthusiasts.
PASSAGE V
Mother Jones: True to the Spirit of Her Cause
The autobiography by Mary Harris Jones is riddled
with factual inaccurate. Jones even fudges her date of
birth, she falsely lists May 1, International Workers
Day, and ages herself by nearly a decade. These
untruthswhether deliberate exaggerations or
slips of the memoryultimately matters very
little, for the autobiography isn’t about the life of
Mary Harris Jones. Jones became famous for her work.
58
59
61
62
63
64
65
When Mary Harris Jones got involved
in labor politics in the 1860s, it was rare for
a woman to attend, let alone address, union
meetings. Jones, however, became one of the
movements most powerful and controversial advocates.
She traveled the United States, from the coal mines of
Appalachia to the railroad yards of the West, rallying
workers to join unions and fight for better working
conditions. Specifically, Jones helped organize efforts
to ensure that employers complied with laws governing
workday hours and child labor.
The moniker Mother Jones was conferred on Jones
by members of the American Railway Union. She herself,
adopted the name and, subsequently, a corresponding
public persona. Her audiences came to expect Mother
Jones. d By 1900, the white-haired, calico-frocked
figure was no longer known as Mary Harris Jones,
the media, union leaders and workers, and even U.S.
presidents referred to her as Mother Jones.
Embracing the very role used to confine
women to the domestic sphere, Jones subversively
redefined the boundaries of home and family.
66. F. NO CHANGE
G. movement’s most powerful and controversial
advocates.
H. movement’s most powerful and controversial
advocates.
J. movements most powerful and controversial
advocates.
67. A. NO CHANGE
B. She, herself,
C. She, herself
D. She herself
68. At this point, the writer is considering adding the fol-
lowing true statement:
To meet their expectations, Jones crafted her
speech, dress, and mannerisms based on cul-
tural notions of motherhood.
Should the writer make this addition here?
F. Yes, because it highlights the contrast between
Jones’s personal style and her audiences.
G. Yes, because it adds details about what types of
changes Jones made to create her public persona.
H. No, because it detracts from the focus of the para-
graph by introducing unrelated details.
J. No, because it doesnt indicate the effect Jones’s
public persona had on audiences.
69. A. NO CHANGE
B. Jones, in fact,
C. Jones in fact
D. Jones;
GO ON TO THE NEXT PAGE.
ACT-1572CPRE
11
66
67
69
22
23
70. If the writer were to delete the underlined portion, the
paragraph would primarily lose a quotation that:
F. questions the distinction between Mary Harris
Jones and her public persona, Mother Jones.
G. reinforces the essays characterization of Mother
Jones as a happy-go-lucky vagabond.
H. reiterates the point that Jones enjoyed the travel
opportunities her work provided.
J. provides support for the claim that Jones redefined
the boundaries of home.
71. In the preceding sentence, the writer is considering
replacing workers with her family of workers.”
Should the writer make this revision?
A. Yes, because it completes the metaphor comparing
Jones to the head of a family.
B. Yes, because it makes clear that Jones cared most
about workers who were family relatives.
C. No, because it unnecessarily repeats information
established earlier in the essay.
D. No, because it introduces an unrelated comparison
between workers and family.
72. F. NO CHANGE
G. protections, to name a few, included:
H. she defined protection as:
J. she did this by:
73. A. NO CHANGE
B. Because of
C. Without
D. Despite
74. F. NO CHANGE
G. they’re behalves,
H. their behalf,
J. their behalves,
75. Suppose the writer’s goal had been to summarize
womens contributions to early-twentieth-century labor
law reform. Would this essay accomplish that goal?
A. Yes, because it shows that Mother Jones was a
well-known and respected labor agitator.
B. Yes, because it introduces a prominent figure in
labor history.
C. No, because it focuses more specifically on labor
law reform in the nineteenth century.
D. No, because it focuses more specifically on one
figure in the labor movement.
ACT-1572CPRE
11
“My address is like my shoes, she said. “It travels with
me wherever I go. She was the matriarch who staunchly
protected workers. g
And protect them she did: When workers
went on strike, Jones secured food donations and
temporary living arrangements. Where companies
prevented the formation of unions, she fought for
workers right to organize. Instead of these tireless
efforts on there behalf, workers trusted Mother Jones
and, by extension, the labor unions she represented.
Question 75 asks about the preceding passage
as a whole.
END OF TEST 1
STOP! DO NOT TURN THE PAGE UNTIL TOLD TO DO SO.
70
72
73
74
70
1. The blood types of 150 people were determined for a
study as shown in the figure below.
If 1person from this study is randomly selected, what
is the probability that this person has either TypeA or
TypeAB blood?
A.
B.
C.
D.
E.
2. The monthly fees for single rooms at 5 colleges are
$370, $310, $380, $340, and $310, respectively. What
is the mean of these monthly fees?
F. $310
G. $340
H. $342
J. $350
K. $380
62
____
150
66
____
150
68
____
150
73
____
150
84
____
150
O B AB
number of people
blood type
70
60
50
40
30
20
10
0
62
67
15
6
A
3. On a particular road map, inchrepresents 18 miles.
About how many miles apart are 2 towns that are
2 inches apart on this map?
A. 18
B. 22
C. 36
D. 45
E. 90
4. Given f = cd
3
, f = 450, and d = 10, what is c ?
F.
000.45
G. 004.5
H. 015
J. 045
K. 150
5. If f (x)= (3x + 7)
2
, then f (1)= ?
A. 010
B. 016
C. 058
D. 079
E. 100
6. Jorge’s current hourly wage for working at
Denti Smiles is $12.00. Jorge was told that at the
beginning of next month, his new hourly wage will be
an increase of 6% of his current hourly wage. What
will be Jorges new hourly wage?
F. $12.06
G. $12.60
H. $12.72
J. $18.00
K. $19.20
1
__
2
1
__
2
1
__
2
GO ON TO THE NEXT PAGE.
MATHEMATICS TEST
60 Minutes—60 Questions
ACT-1572CPRE
22
DIRECTIONS: Solve each problem, choose the correct
answer, and then ll in the corresponding oval on your
answer document.
Do not linger over problems that take too much time.
Solve as many as you can; then return to the others in
the time you have left for this test.
You are permitted to use a calculator on this test. You
may use your calculator for any problems you choose,
but some of the problems may best be done without
using a calculator.
Note: Unless otherwise stated, all of the following should
be assumed.
1. Illustrative gures are NOT necessarily drawn to scale.
2. Geometric gures lie in a plane.
3. The word line indicates a straight line.
4. The word average indicates arithmetic mean.
24
25
7. The first term is 1 in the geometric sequence
1, 3, 9, 27,
. What is the SEVENTH term of the
geometric sequence?
A. 243
B.
030
C. 081
D. 189
E. 729
8. The shipping rate for customers of Ship Quick consists
of a fee per box and a price per pound for each box.
The table below gives the fee and the price per pound
for customers shipping boxes of various weights.
Gregg wants Ship Quick to ship 1 box that weighs
15pounds. What is the shipping rate for this box?
F. $09.75
G. $16.50
H. $19.75
J. $20.00
K. $24.50
9. A computer chip 0.32 cm thick is made up of layers of
silicon. If the top and bottom layers are each 0.03 cm
thick and the inner layers are each 0.02 cm thick, how
many inner layers are there?
A. 13
B. 15
C. 16
D. 52
E. 64
10. The table below shows the number of cars Jing sold
each month last year. What is the median of the data in
the table?
F. 13
G. 16
H. 19
J. 20.5
K. 23.5
Weight of box
(pounds) Fee Price per pound
Less than 10
1025
More than 25
$05.00
$10.00
$20.00
$1.00
$0.65
$0.30
0.32 cm
}
Month Number of cars sold
January 25
February 15
March 22
April 19
May 16
June 13
July 19
August 25
September 26
October 27
November 28
December 29
11. Students studying motion observed a cart rolling at a
constant rate along a straight line. The table below
gives the distance, d feet, the cart was from a reference
point at 1-second intervals from t = 0 seconds to
t = 5seconds.
Which of the following equations represents this
relationship between d and t ?
A. d= 00t + 14
B. d= 06t + 08
C. d= 06t + 14
D. d= 14t + 06
E. d= 34t
12. The length of a rectangle with area 54 square
centimeters is 9 centimeters. What is the perimeter of
the rectangle, in centimeters?
F. 06
G. 12
H. 15
J. 24
K. 30
13. In the figure below, C is the intersection of AD
___
and
BE
___
. If it can be determined, what is the measure of
BAC ?
A. 080°
B. 100°
C. 110°
D. 115°
E. Cannot be determined from the given information
14. Antwan drew the circle graph below describing his
time spent at school in 1day. His teacher said that the
numbers of hours listed were correct, but that the
central angle measures for the sectors were not correct.
What should be the central angle measure for the Core
subjects sector?
F.
072°
G. 080°
H. 160°
J. 200°
K. 288°
t
00 01 02 03 04 05
d 14 20 26 32 38 44
35°
35°45°
B
C
D
E
A
Core
subjects
4 hours
Electives
3 hours
Lunch
and
passing time
1 hour
Choir
1 hour
GO ON TO THE NEXT PAGE.
ACT-1572CPRE
22
20. For trapezoid ABCD shown below, AB
___
i
DC
____
, the
measures of the interior angles are distinct, and the
measure of D is x°. What is the degree measure of
A in terms of x ?
F. (180 x)°
G. (180 0.5x)°
H. (180+ 0.5x)°
J. (180+ x)°
K. x°
21. To get a driver’s license, an applicant must pass a
written test and a driving test. Past records show that
80% of the applicants pass the written test and 60% of
those who have passed the written test pass the driving
test. Based on these figures, how many applicants in a
random group of 1,000applicants would you expect to
get driver’s licenses?
A. 200
B. 480
C. 600
D. 750
E. 800
22. If a, b, and c are positive integers such that a
b
= x and
c
b
= y, then xy = ?
F. ac
b
G. ac
2b
H. (ac)
b
J. (ac)
2b
K. (ac)
b
2
23. Which of the following expressions is equivalent to
y
2
(6x + 2y + 12x 2y)?
A.
09xy
2
B. 18xy
C. 03xy
2
+ 12x
D. 09xy
2
2y
3
E. 03xy
2
+ 12x y
3
2y
24. An artist makes a profit of (500p p
2
) dollars from
selling p paintings. What is the fewest number of
paintings the artist can sell to make a profit of at least
$60,000?
F. 100
G. 150
H. 200
J. 300
K. 600
x°
1
__
2
GO ON TO THE NEXT PAGE.
ACT-1572CPRE
22
15. This month, Kami sold 70 figurines in 2 sizes. The
large figurines sold for $12 each, and the small
figurines sold for $8 each. The amount of money he
received from the sales of the large figurines was equal
to the amount of money he received from the sales of
the small figurines. How many large figurines did
Kami sell this month?
A. 20
B. 28
C. 35
D. 42
E. 50
16. A car accelerated from 88 feet per second (fps) to
220 fps in exactly 3 seconds. Assuming the acceleration
was constant, what was the cars acceleration, in feet
per second per second, from 88fps to 220fps?
F. 00
G. 029
H. 044
J. 075
K. 102
17. In a plane, the distinct lines AB
and CD
intersect at A,
where A is between C and D. The measure of BAC is
47°. What is the measure of BAD ?
A. 043°
B. 047°
C. 094°
D. 133°
E. 137°
18. In which of the following are , , and arranged in
ascending order?
F. <<
G. <<
H. <<
J. <<
K. <<
19. In scientific notation, 670,000,000+ 700,000,000= ?
A. 1.37
×
10
9
B. 1.37
×
10
7
C. 1.37
×
10
8
D. 1.37
×
10
9
E. 137
×
10
15
1
___
44
2
__
3
1
__
3
1
__
3
5
__
8
5
__
6
1
__
2
5
__
6
5
__
8
1
__
2
5
__
8
1
__
2
5
__
6
1
__
2
5
__
8
5
__
6
5
__
6
1
__
2
5
__
8
1
__
2
5
__
6
5
__
8
26
27
29. What is the product of the complex numbers (3i + 4)
and (3i + 4)?
A.
01
B. 07
C. 25
D. 7+ 24i
E. 07+ 24i
30. The radius of the base of the right circular cone shown
below is 5 inches, and the height of the cone is
7 inches. Solving which of the following equations
gives the measure, , of the angle formed by a slant
height of the cone and a radius?
F. tan, =
G. tan, =
H. sin, =
J. sin, =
K. cos,=
31. To make a 750-piece jigsaw puzzle more challenging, a
puzzle company includes 5 extra pieces in the box
along with the 750pieces, and those 5extra pieces do
not fit anywhere in the puzzle. If you buy such a puzzle
box, break the seal on the box, and immediately select
1 piece at random, what is the probability that it will
be 1of the extra pieces?
A.
B.
C.
D.
E.
32. What fraction lies exactly halfway between and ?
F.
G.
H.
J.
K.
7
5
θ
5
__
7
7
__
5
5
__
7
7
__
5
7
__
5
1
__
5
1
____
755
1
____
750
5
____
755
5
____
750
3
__
4
2
__
3
3
__
5
5
__
6
7
___
12
9
___
16
17
___
24
GO ON TO THE NEXT PAGE.
ACT-1572CPRE
22
25. Last month, Lucie had total expenditures of $900. The
pie chart below breaks down these expenditures by
category. The category in which Lucie’s expenditures
were greatest is what percent of her total expenditures,
to the nearest 1%?
A. 24%
B. 28%
C. 32%
D. 34%
E. 39%
26. In the figure shown below, the measure of BAC is
(x + 20)° and the measure of BAD is 90°. What is the
measure of CAD ?
F. 0(x 70)°
G. 0(70 x)°
H. 0(70+ x)°
J. (160 x)°
K. (160+ x)°
27. What is the perimeter, in inches, of the isosceles right
triangle shown below, whose hypotenuse is 8
å
2 inches
long?
A. 08
B. 08+ 08
å
2
C. 08+ 16
å
2
D. 16
E. 16+ 08
å
2
28. The equation y = ax
2
+ bx + c is graphed in the
standard (x,y) coordinate plane below for real values of
a, b, and c. When y = 0, which of the following best
describes the solutions for x ?
F. 2distinct positive real solutions
G. 2distinct negative real solutions
H. 1positive real solution and 1negative real solution
J. 2real solutions that are not distinct
K. 2distinct solutions that are not real
entertainment
$125
$219
food
$254
clothes
$120
gas
$182
i
nsurance
A
C
D
B
O
x
y
36. Which of the following is the graph of the region
1<x + y <2 in the standard (x,y) coordinate plane?
37. What is the difference between the mean and the
median of the set {3, 8, 10, 15}?
A. 00
B. 01
C. 04
D. 09
E. 12
38. Which of the following describes a true relationship
between the functions f (x) = (x 3)
2
+ 2 and
g(x) = x + 1 graphed below in the standard (x,y)
coordinate plane?
F. f(x) = g(x) for exactly 2values of x
G. f(x) = g(x) for exactly 1 value of x
H. f(x) < g(x) for all x
J. f (x) > g(x) for all x
K. f(x) is the inverse of g(x)
F
.
2
1
1
2
y
x
y
G
.
H
.
2
1
1
2
x
J.
2
1
1
2
y
x
K.
2
1
1
2
y
x
2
1
12
y
x
y
x
O
1
__
2
GO ON TO THE NEXT PAGE.
ACT-1572CPRE
22
Gianna is converting a 12-foot-by-15-foot room in her
house to a craft room. Gianna will install tile herself but
will have CC Installations build and install the cabinets.
The scale drawing shown below displays the location of the
cabinets in the craft room (0.25inch represents 2feet).
Cabinets will be installed along one of the 12-foot walls
from floor to ceiling, and 4cabinets that are each 3feet tall
will be installed in the middle of the room. These are the
only cabinets that will be installed, and each of them will
be 2 feet wide and 2 feet deep. CC Installations has given
Gianna an estimate of $2,150.00 for building and installing
the cabinets.
33. A 15-foot wall is how many inches long in the scale
drawing?
A. 1.5
B. 1.875
C. 3
D. 3.375
E. 3.75
34. Gianna will install tile on the portion of the floor that
will NOT be covered by cabinets. What is the area, in
square feet, of the portion of the floor that will NOT be
covered by cabinets?
F. 072
G. 090
H. 140
J. 156
K. 164
35. CCInstallations estimate consists of a $650.00charge
for labor, plus a fixed charge per cabinet. The labor
charge and the charge per cabinet remain the same
for any number of cabinets built and installed.
CC Installations would give Gianna what estimate if
the craft room were to have twice as many cabinets as
Gianna is planning to have?
A. $2,800.00
B. $3,000.00
C. $3,450.00
D. $3,650.00
E. $4,300.00
window
cabinets
door
2 ft deep
15 ft wall
12 ft wal
l
cabinets
Use the following information to answer
questions 3335.
28
29
43. A formula to estimate the monthly payment, p dollars,
on a short-term loan is
p =
where a dollars is the amount of the loan, r is the
annual interest rate expressed as a decimal, and y years
is the length of the loan. When a is multiplied by 2,
what is the effect on p ?
A. p is divided by 6
B. p is divided by 2
C. p does not change
D. p is multiplied by 2
E. p is multiplied by 4
44. The pointsE(6,4) and F(14,12) lie in the standard (x,y)
coordinate plane shown below. Point D lies on EF
___
between E and F such that the length of EF
___
is 4times
the length of DE
___
. What are the coordinates of D ?
F. (
07,05)
G. (08,06)
H. (08,08)
J. (10,08)
K. (12,10)
45. Given that a
34
=
34
for some real number a,
what is x + z ?
A.
B.
C. 26
D. 27
E. 48
46. A container is full of water. After 10 cups of water
are added, the container is full. What is the volume
of the container, in cups?
F. 13
G. 13
H. 15
J. 16
K. 40
1
__
2
ary + a
_________
12y
y
x
O
E(6,4)
F(14,12
)
27
z
x
y
6
4
2
1
4
__
3
27
___
2
1
__
8
3
__
4
1
__
3
1
__
2
GO ON TO THE NEXT PAGE.
ACT-1572CPRE
22
Trapezoid ABCD is graphed in the standard (x,y) coordinate
plane below.
39. What is the slope of CD
___
?
A. 3
B. 1
C. 1
D.
E.
40. When ABCD is reflected over the y-axis to ABCD,
what are the coordinates of D ?
F. (12,0 1)
G. (12,01)
H. ( 12,01)
J. ( 01, 12)
K. ( 01,12)
41. Which of the following vertical lines cuts ABCD into
2trapezoids with equal areas?
A. x= 2.5
B. x= 3.5
C. x= 4.5
D. x= 5.5
E. x= 6.5
42. Given f (x)= x and g(x)= ,what is f
1
g
12
2
?
F. 3
G.
H.
J. 0
K.
O
y
x
B(3,4) C(9,4)
A(2,1)
D(12,1)
5
___
21
3
__
2
1
__
2
1
__
x
1
__
x
3
__
2
2
__
3
3
__
2
Use the following information to answer
questions 3941.
50. You can find the volume of an irregularly shaped solid
object by completely submerging it in water and
calculating the volume of water the object displaces.
You completely submerge a solid object in a
rectangular tank that has a base 40 centimeters by
30 centimeters and is filled with water to a depth of
20 centimeters. The object sinks to the bottom, and the
water level goes up 0.25 centimeters. What is the
volume, in cubic centimeters, of the object?
F. 300
G. 240
H. 200
J. 150
K. 075
51. If x:y = 5:2 and y:z = 3:2, what is the ratio of x:z ?
A. 03:1
B. 03:5
C. 05:3
D. 08:4
E. 15:4
52. Which of the following is the solution statement for
the inequality shown below?
5< 1 3x < 10
F. 5< x < 10
G. 3< x
H. 3< x < 2
J. 2< x < 3
K. x <−3orx > 2
53. A formula for the surface area (A) of the rectangular
solid shown below is A = 2lw + 2lh + 2wh where l
represents length; w,width; and h,height. By doubling
each of the dimensions (l, w, and h), the surface area
will be multiplied by what factor?
A. 02
B. 04
C. 06
D. 08
E. 12
54. A dog eats 7 cans of food in 3 days. At this rate, how
many cans of food does the dog eat in 3+ d days?
F. + d
G. +
H. +
J. 7 +
K. 7 +
h
l
w
7
__
3
d
__
3
7
__
3
7
_ __
3d
7
__
3
d
__
3
7d
_ __
3
GO ON TO THE NEXT PAGE.
ACT-1572CPRE
22
47. Only tenth-, eleventh-, and twelfth-grade students
attend Washington High School. The ratio of tenth
graders to the school’s total student population is
86:255, and the ratio of eleventh graders to the
school’s total student population is 18:51. If 1 student
is chosen at random from the entire school, which
grade is that student most likely to be in?
A. Tenth
B. Eleventh
C. Twelfth
D. All grades are equally likely.
E. Cannot be determined from the given information
48. +=?
F.
G.
H.
J.
K.
49. The shaded region in the graph below represents the
solution set to which of the following systems of
inequalities?
A.
5
y <−x + 2
(x 1)
2
+ (y 2)
2
< 9
B.
5
y >−x + 2
(x 1)
2
+ (y 2)
2
< 9
C.
5
y >−x + 2
(x 1)
2
+ (y 2)
2
> 9
D.
5
y <−x + 2
(x 1)
2
+ (y 2)
2
> 9
E.
5
(y 2) < 3
(x 1) > 3
y
x
(x 1 )
2
+ (y 2)
2
=
9
y = x + 2
4
å
3 + 2
å
2
____________
å
5
4
å
3 + 2
å
2
____________
å
6
6
_________
å
2 +
å
3
6
____
å
5
8
____
å
6
2
____
å
3
4
____
å
2
30
31
55. Kelly asked 120 students questions about skiing. The
results of the poll are shown in the table below.
After completing the poll, Kelly wondered how many
of the students polled had skied both cross-country and
downhill. How many of the students polled indicated
that they had skied both cross-country and downhill?
A. 73
B. 65
C. 47
D. 18
E.
08
56. The square below is divided into 3 rows of equal area.
In the top row, the region labeled A has the same area
as the region labeled B. In the middle row, the 3 regions
have equal areas. In the bottom row, the 4 regions have
equal areas. What fraction of the square’s area is in a
region labeled A?
F.
G.
H.
J.
K.
57. The functions y = sin,x and y = sin(x + a) + b, for
constants a and b, are graphed in the standard (x,y)
coordinate plane below. The functions have the same
maximum value. One of the following statements
about the values of a and b is true. Which statement is
it?
A. a< 0 and b= 0
B. a< 0 and b> 0
C. a= 0 and b> 0
D. a> 0 and b< 0
E. a> 0 and b> 0
Question Yes No
1. Have you skied either cross-country
or downhill?
65 55
2. If you answered Yes to Question1,
did you ski downhill?
28 37
3. If you answered Yes to Question1,
did you ski cross-country?
45 20
BA
C
CD
BA
BA
1
__
9
3
__
9
6
__
9
13
___
12
13
___
36
O
y
x
58. Which of the following number line graphs shows the
solution set to the inequality
x 5
<−1?
59. As part of a probability experiment, Elliott is to answer
4 multiple-choice questions. For each question, there
are 3 possible answers, only 1 of which is correct. If
Elliott randomly and independently answers each
question, what is the probability that he will answer
the 4questions correctly?
A.
B.
C.
D.
E.
60. The sides of an acute triangle measure 14 cm, 18 cm,
and 20 cm, respectively. Which of the following
equations, when solved for , gives the measure of the
smallest angle of the triangle?
(Note: For any triangle with sides of length a, b, and c
that are opposite angles A, B, and C, respectively,
==and c
2
= a
2
+ b
2
2ab cos
,C.)
F. =
G. =
H. =
J. 14
2
= 18
2
+ 20
2
2(18)(20)cos,
K. 20
2
= 14
2
+ 18
2
2(14)(18)cos,
F.
G
.
4
x
H
.
6
x
J
.
6
4
x
K
.
6
4
x
x
(empty set)
6
4
6
4
27
___
81
12
___
81
4
___
81
3
___
81
1
___
81
sin
,C
_____
c
sin,B
_____
b
sin,A
_____
a
1
___
18
sin
,
_____
14
1
___
20
sin
,
_____
14
1
___
14
sin
,
_____
20
ACT-1572CPRE
22
END OF TEST 2
STOP! DO NOT TURN THE PAGE UNTIL TOLD TO DO SO.
DO NOT RETURN TO THE PREVIOUS TEST.
Passage I
PROSE FICTION: Th pa age adap ed from the novel The
Ground Beneath Her Feet by Salman Rushdie1999 by
Salman Rushdie).
Art Deco is an architectural and decorative style that was popu-
lar in the first half of the twentieth century.
When you grow up, as I did, in a great city, during
what just happens to be its golden age, you think of it
as eternal. Always was there, always will be. The
grandeur of the metropolis creates the illusion of per-
manence. The peninsular Bombay into which I was
born certainly seemed perennial to me. Malabar and
Cumballa hills were our Capitol and Palatine, the
Brabourne Stadium was our Colosseum, and as for the
glittering Art Deco sweep of Marine Drive, well, that
was something not even Rome could boast. I actually
grew up believing Art Deco to be the Bombay style,” a
local invention, its name derived, in all probability,
from the imperative of the verb to see. Art dekho. Lo
and behold art. (When I began to be familiar with
images of New York, I at first felt a sort of anger. The
Americans had so much; did they have to possess our
“style as well? But in another, more secret part of my
heart, the Art Deco of Manhattan, built on a scale so
much grander than our own, only increased Americas
allure, made it both familiar and awe-inspiring, our
little Bombay writ large.)
In reality that Bombay was almost brand-new
when I knew it; what’s more, my parents’ construction
firm of Merchant & Merchant had been prominent in its
making. In the ten years before my own coming into the
world, the city had been a gigantic building site; as if it
were in a hurry to become, as if it knew it had to pro-
vide itself in finished condition by the time I was able
to start paying attention to it ... No, no, I don’t really
think along such solipsistic lines. Im not over-attached
to history, or Bombay. Me, Im the under-attached type.
But let me confess that, even as a child, I was
insanely jealous of the city in which I was raised,
because it was my parentsother love. They loved each
other (good), they loved me (very good), and they
loved her (not so good). Bombay was my rival. It was
on account of their romance with the city that they
drew up that weekly rota (list) of shared parental
responsibilities. When my mother wasnt with me
when I was riding on my fathers shoulders, or staring,
with him, at the fish in the Taraporewala Aquarium
she was out there with her, with Bombay; out there
bringing her into being. (For of course construction
work never stops completely, and supervising such
work was Ameer’s particular genius. My mother the
master builder. Like her father before her.) And when
my father handed me over to her, he went off, wearing
his local-history hat and a khaki jacket full of pockets,
to dig in the foundations of building sites for the secrets
of the city’s past, or else sat hatless and coatless at a
designing board and dreamed his lo-and-behold dreams.
Maps of the early town afforded my father great
joy, and his collection of old photographs of the edi-
fices and objets of the vanished city was second to
none. In these faded images were resurrected the
demolished Fort, the breakfast bazaar market outside
the Teen Darvaza or Bazaargate, and the humble mutton
shops and umbrella hospitals of the poor, as well as the
fallen palaces of the great. The early city’s relics filled
his imagination as well as his photo albums. It was
from my father that I learned of Bombay’s first great
photographers, Raja Deen Dayal and A. R. Haseler,
whose portraits of the city became my first artistic
influences, if only by showing me what I did not want
to do. Dayal climbed the Rajabai tower to create his
sweeping panoramas of the birth of the city; Haseler
went one better and took to the air. Their images were
awe-inspiring, unforgettable, but they also inspired in
me a desperate need to get back down to ground level.
From the heights you see only pinnacles. I yearned for
the city streets, the knife grinders, the water carriers,
the pavement moneylenders, the peremptory soldiers,
the railway hordes, the chess players in the Irani restau-
rants, the snake-buckled schoolchildren, the beggars,
the fishermen, the moviemakers, the dockers, the book
sewers, the loom operators, the priests. I yearned for
life.
When I said this to my father he showed me
photos, still lives of storefronts and piers, and told me I
was too young to understand. “See where people lived
and worked and shopped, he clarified, with a rare flash
of irritation, and it becomes plain what they were
like. For all his digging, Vivvy Merchant was content
with the surfaces of his world. I, his photographer son,
set out to prove him wrong, to show that a camera can
see beyond the surface, beyond the trappings of the
actual, and penetrate to its flesh and heart.
READING TEST
35 Minutes—40 Questions
DIRECTIONS: There are several passages in this test.
Each passage is accompanied by several questions.
After reading a passage, choose the best answer to each
question and fill in the corresponding oval on your
answer document. You may refer to the passages as
often as necessary.
GO ON TO THE NEXT PAGE.
ACT-1572CPRE
33
5
10
15
20
25
30
35
40
45
50
55
60
65
70
75
80
85
32
33
1. The passage as a whole can primarily be characterized
as the narrator’s:
A. explanation of the relationship the narrator and his
parents had with the city of Bombay.
B. description of important buildings and locations in
Bombay.
C. argument for Bombay’s prominence in the world
of architecture.
D. concerns about the emotional environment in
which the narrator was raised.
2. The narrator describes the photos by Bombay’s first
great photographers as primarily inspiring the narrator
to:
F. turn away from a career in photography.
G. create grand panoramas of the new Bombay.
H. produce images that his father would add to his
collection.
J. photograph subjects that depict everyday life on
Bombay’s streets.
3. In lines 2531, the narrator muses over, then rejects,
the notion that:
A. Merchant & Merchant played an important role in
the building of Bombay.
B. he started paying attention to Bombay at a young
age.
C. his anticipated birth was one of the causes of the
rush to finish the building of Bombay.
D. Bombay had been a gigantic building site in the
years before he was born.
4. In lines 3243, the narrator uses which of the follow-
ing literary devices to describe Bombay?
F. Alliteration
G. Allusion
H. Personification
J. Simile
5. Which of the following statements best captures how
the narrator’s parents balanced their parental duties
with their work at the construction company?
A. The narrators mother did the majority of the work
at the construction company, while the narrator’s
father took care of the narrator.
B. The narrator’s parents traded off responsibility for
taking care of the narrator and working at the con-
struction company.
C. The narrator’s father worked at his designing
board, while the narrator’s mother took the narra-
tor along to building sites.
D. The narrator’s parents both worked at the con-
struction company, while the narrator stayed home
with a babysitter.
6. As it is used in line 9, the word sweep most nearly
means:
F. overwhelming victory.
G. wide-ranging search.
H. complete removal.
J. broad area.
7. In the context of the passage, the primary function of
lines 610 is to:
A. compare architectural landmarks in Bombay to
those elsewhere.
B. help illustrate how the term art deco was
derived.
C. contradict the idea that Bombay was in its golden
age when the narrator was a child.
D. provide examples of “Bombay style architecture
in Rome.
8. The narrator as a child viewed the work his parents did
for Merchant & Merchant with a strong sense of:
F. joy; the work provided the family with enough
money to live extravagant lives.
G. fear; the narrator knew his parents were often so
exhausted they were careless about safety.
H. jealousy; the work pulled the narrator’s parents
away from him and directed their attention to the
city.
J. respect; his parents were known for their quality
workmanship throughout the city.
9. As it is used in line 38, the phrase drew up most nearly
means:
A. extended.
B. prepared.
C. approached.
D. straightened.
10. In the last paragraph, the narrators father shows the
narrator the photos of storefronts and piers in order to:
F. teach the narrator about the commercial progress
the people who work in Bombay have made.
G. convince the narrator that Dayal and Haseler were
Bombay’s first great photographers.
H. clarify his claim that his photo collection was not
about modern-day Bombay but rather about the
early twentieth century.
J. illustrate that photos of places can reveal as much
about the people who spent time there as photos of
the people themselves.
GO ON TO THE NEXT PAGE.
ACT-1572CPRE
33
Passage II
SOCIAL SCIENCE: This passage is adapted from Great
Waters: An Atlantic Passage by Deborah Cramer (©2001 by
Deborah Cramer).
The Sargasso Sea is a part of the northern Atlantic Ocean.
As the Cramer idles through the Sargasso Sea,
waiting for the wind to rise, the sea is flat and empty.
Nothing demarcates or divides the smooth expanse of
water dissolving into the horizon. This vast, unrough-
ened surface, this breadth of uniform sea, deceives. But
for a few lonely oceanic islands, the unperturbed sur-
face offers no hint of the grand and sweeping energies
hidden below.
Only one thousand miles offshore, the Cramer has
already sailed through some of Atlantic’s deepest
waters. Contrary to what one might guess, Atlantic’s
deepest waters, like those in other oceans, are along her
edges. As we continue east, toward the middle of the
sea, the bottom rises. The unmarked plains of the abyss,
here flattened by layers of sediment, give way to rising
foothills and then to mountains. The first maps of
Atlantic seafloor noted, albeit crudely, this rise. Early
efforts to plumb Atlantic’s depths proved outrageously
inaccurate: one naval officer paid out eight miles (thir-
teen kilometers) of hemp rope from a drifting ship and
concluded the sea had no bottom. Eventually, sailors
more or less successfully calculated depth by heaving
overboard cannonballs tied to bailing twine. When they
hit bottom, the sailors measured and snipped the twine
and then moved on, leaving a trail of lead strung out
across the seafloor. These crude soundings, forming the
basis of the first map of Atlantics basin, published in
1854, identified a prominent rise halfway between
Europe and America.
For many years no one could explain why the
basin of Atlantic, unlike a bowl, deepened at its edges
and shoaled in its center. People assumed that this
Middle Ground,” Telegraph Plateau,” or Dolphin
Rise,” as it was variously called, was an ancient and
drowned land bridge, or a lost continent, but sailors
repairing transatlantic telegraph cable unknowingly
produced evidence to prove otherwise. Wrestling with
the broken cable, they accidentally twisted off a piece
of the plateau and dredged up a twenty-one-pound
(ten-kilogram) chunk of dense black volcanic rock. It
was some of the youngest, freshest rock on earth, and it
was torn not from a piece of continent sunk beneath the
waves, but from the very foundation of the sea.
Today, highly sophisticated sound waves bring the
hazy images of those early soundings into sharp focus,
revealing that one of the largest and most salient geo-
graphic features on the planet lies on the floor of the
ocean. Hidden beneath the waves is an immense sub-
merged mountain range, the backbone of the sea. More
extensive, rugged, and imposing than the Andes, Rock-
ies, or Himalayas, it covers almost as much of earths
surface as the dry land of continents. Winding like the
seam of a baseball, it circles the planet in a long, sinu-
ous path, running the entire length of Atlantic, slashing
the basin neatly in two. Its mountains are stark and
black, as black as the sea itself, lit only at their peaks
by a thin, patchy covering of white, the skeletal remains
of tiny microscopic animals that once lived at the sur-
face. Peaks as high as Mount St. Helens sit in a watery
world of blackness, more than a mile below the surface,
beyond the reach of light, beyond the sight of sailors.
A great valley, eclipsing any comparable feature
on dry land, runs through these mountains. Arizona’s
Grand Canyon, one of earths most spectacular places,
extends for about 280 miles (450 kilometers). A lesser-
known canyon of similar depth but considerably greater
length lies hidden in the mountains of the ridge.
Although offset in many places by breaks in the moun-
tains, the rift valley, as the canyon is called, extends the
length of Atlantic for 11,000 miles (17,700 kilometers).
Here in this bleak and forbidding place, where the
water is almost freezing, subterranean fires have lifted
mounds of fresh lava onto the seafloor. Scientists visit-
ing the rift valley for the first time named the volcanic
hills in this otherworldly setting after distant, lifeless
planets.
Yet, what had seemed so foreign to scientists is an
integral part of earth’s very being, for at the ridge our
own planet gives birth. The floor of the rift valley is
torn; from the gashes has sprung the seafloor underly-
ing all of Atlantic. Here the youngest, newest pieces are
made. Earth is still cooling from her tumultuous birth
four and a half billion years ago. Heat, leaking from the
molten core and from radioactive decay deep inside the
planet, rises toward earth’s surface, powering the volca-
noes that deliver the ridge to the sea.
11. The author’s attitude toward the main subject of the
passage can best be described as:
A. awe and fascination.
B. disbelief and cynicism.
C. amusement and nostalgia.
D. boredom and indifference.
12. The passage makes clear that Middle Ground, Tele-
graph Plateau, and Dolphin Rise were names that
people gave to what was actually:
F. an island in Atlantic.
G. a transatlantic telegraph cable.
H. an ancient and drowned land bridge.
J. the immense mountain range in Atlantics basin.
GO ON TO THE NEXT PAGE.
ACT-1572CPRE
33
5
10
15
20
25
30
35
40
45
50
55
60
65
70
75
80
85
34
35
13. In the first paragraph, the author describes the stillness
of the Sargasso Sea as the Cramer passes through it
primarily to emphasize that the stillness:
A. wont last long, for the sea will become rough
when the wind rises.
B. makes it easy for a passenger on the Cramer to
spot oceanic islands that break the water’s surface.
C. is in dramatic contrast to the power of what exists
on and under the seafloor far below.
D. makes it seem as if the Cramers wake is dividing
the unbroken expanse of water into two.
14. The passage states that compared to Arizona’s Grand
Canyon, the canyon that lies within the mountains in
Atlantic’s basin is considerably:
F. deeper.
G. older.
H. wider.
J. longer.
15. The main purpose of the information in lines 7176 is
to:
A. describe in detail scientists expectations for their
first trip to the rift valley.
B. characterize the rift valley as an alien, seemingly
barren place.
C. provide statistics about several geographic proper-
ties of the rift valley.
D. list the names that scientists gave to the volcanic
hills in the rift valley.
16. One of the main purposes of the last paragraph is to
state that the:
F. gashes in the rift valley continue to increase in
width.
G. seafloor of Atlantic has cooled.
H. entire Atlantic seafloor has issued from the gashes
in the rift valley.
J. volcanoes on Earth’s dry land have created the
newest, youngest pieces of Atlantic seafloor.
17. The author most strongly implies that people com-
monly assume the deepest waters of an ocean are:
A. about one thousand miles offshore.
B. at the middle of the ocean.
C. dotted with islands.
D. located in trenches.
18. As it is used in line 19, the phrase paid out most nearly
means:
F. dispensed.
G. ascertained.
H. suggested.
J. compensated.
19. According to the passage, the mountain range in
Atlantic’s basin covers nearly the same amount of
Earth’s surface as does:
A. Mount St. Helens.
B. the Himalayas.
C. the Pacific Ocean.
D. the dry land of continents.
20. According to the passage, the white cover on the peaks
of the mountains in Atlantic’s basin is:
F. skeletal remains of microscopic animals.
G. thin layers of sedimentary volcanic ash.
H. patches of ice.
J. salt deposits.
GO ON TO THE NEXT PAGE.
ACT-1572CPRE
33
GO ON TO THE NEXT PAGE.
ACT-1572CPRE
33
Passage III
HUMANITIES: Passage A is adapted from the essay Just This
Side of Byzantiumby Ray Bradbury (©1975 by Ray Bradbury),
which is the introduction to a later edition of Bradbury’s 1957
novel Dandelion Wine. Passage B is adapted from Dandelion
Wine 1957 by Ray Bradbury).
Passage A by Ray Bradbury
I began to learn the nature of surprises, thankfully,
when I was fairly young as a writer. Before that, like
every beginner, I thought you could beat, pummel, and
thrash an idea into existence. Under such treatment, of
course, any decent idea folds up its paws, turns on its
back, fixes its eyes on eternity, and dies.
It was with great relief, then, that in my early
twenties I floundered into a word-association process in
which I simply got out of bed each morning, walked to
my desk, and put down any word or series of words that
happened along in my head.
I would then take arms against the word, or for it,
and bring on an assortment of characters to weigh the
word and show me its meaning in my own life. An hour
or two hours later, to my amazement, a new story
would be finished and done. The surprise was total and
lovely. I soon found that I would have to work this way
for the rest of my life.
First I rummaged my mind for words that could
describe my personal nightmares, fears of night and
time from my childhood, and shaped stories from these.
Then I took a long look at the green apple trees
and the old house I was born in and the house next door
where lived my grandparents, and all the lawns of the
summers I grew up in, and I began to try words for all
that.
I had to send myself back, with words as catalysts,
to open the memories out and see what they had to
offer.
So from the age of twenty-four to thirty-six hardly
a day passed when I didn’t stroll myself across a recol-
lection of my grandparents northern Illinois grass,
hoping to come across some old half-burnt firecracker,
a rusted toy, or a fragment of letter written to myself in
some young year hoping to contact the older person I
became to remind him of his past, his life, his people,
his joys, and his drenching sorrows.
Along the way I came upon and collided, through
word-association, with old and true friendships. I bor-
rowed my friend John Huff from my childhood in Ari-
zona and shipped him East to Green Town so that I
could say good-bye to him properly.
Along the way, I sat me down to breakfasts,
lunches, and dinners with the long dead and much
loved.
Thus I fell into surprise. I came on the old and best
ways of writing through ignorance and experiment and
was startled when truths leaped out of bushes like quail
before gunshot. I blundered into creativity as any child
learning to walk and see. I learned to let my senses and
my Past tell me all that was somehow true.
Passage B by Ray Bradbury
The facts about John Huff, aged twelve, are simple
and soon stated. He could pathfind more trails than
anyone since time began, could leap from the sky like a
chimpanzee from a vine, could live underwater two
minutes and slide fifty yards downstream from where
you last saw him. The baseballs you pitched him he hit
in the apple trees, knocking down harvests. He ran
laughing. He sat easy. He was not a bully. He was kind.
He knew the names of all the wild flowers and when
the moon would rise and set. He was, in fact, the only
god living in the whole of Green Town, Illinois, during
the twentieth century that Douglas Spaulding knew of.
And right now he and Douglas were hiking out
beyond town on another warm and marble-round day,
the sky blue blown-glass reaching high, the creeks
bright with mirror waters fanning over white stones. It
was a day as perfect as the flame of a candle.
Douglas walked through it thinking it would go on
this way forever. The sound of a good friend whistling
like an oriole, pegging the softball, as you horse-
danced, key-jingled the dusty paths; things were at
hand and would remain.
It was such a fine day and then suddenly a cloud
crossed the sky, covered the sun, and did not move
again.
John Huff had been speaking quietly for several
minutes. Now Douglas stopped on the path and looked
over at him.
“John, say that again.”
You heard me the first time, Doug.
“Did you say you were—going away?”
John took a yellow and green train ticket solemnly
from his pocket and they both looked at it.
Tonight! said Douglas. My gosh! Tonight we
were going to play Red Light, Green Light and Statues!
How come, all of a sudden? You been here in Green
Town all my life. You just don’t pick up and leave!
It’s my father, said John. He’s got a job in Mil-
waukee. We werent sure until today ...
They sat under an old oak tree on the side of the
hill looking back at town. Out beyond, in sunlight, the
town was painted with heat, the windows all gaping.
Douglas wanted to run back in there where the town, by
its very weight, its houses, their bulk, might enclose
and prevent Johns ever getting up and running off.
5
10
15
20
25
30
35
40
45
50
55
60
65
70
75
80
85
90
95
36
37
21. When Bradbury claims, Thus I fell into surprise
(line 46), he’s most nearly referring to the:
A. discovery that for him the secret to a creative out-
pouring was to use a word-association method to
write fiction.
B. long-forgotten experiences he would remember
when he would talk with his childhood friends in
person.
C. realization that he wrote more effectively about his
current experiences than about his past.
D. several methods other writers taught him to help
him write honest, authentic stories.
22. Passage A indicates that Bradbury believes all begin-
ning writers think that they can:
F. learn the nature of surprises.
G. force an idea into creation.
H. use one word as a catalyst for a story.
J. become a good writer through experiment.
23. Bradbury’s claim “I would then take arms against the
word, or for it (line 12) most strongly suggests that
during his writing sessions, Bradbury would:
A. attempt to find the one word that for him was the
key to understanding John Huff.
B. often reject a word as not being a catalyst for
meaningful writing.
C. deliberately choose to write only about a word that
inspired his fears.
D. feel as though he were struggling to find a word’s
significance to him.
24. In the seventh paragraph of Passage A (lines 3037),
Bradbury explains his habit, over many years as a
writer, of almost daily:
F. looking at and writing about objects from his
childhood that he had saved.
G. wishing he had kept more letters from his child-
hood to trigger his memories.
H. driving past his grandparents property, hoping to
notice something that would remind him of his
past.
J. thinking about his grandparents property, hoping
to remember something that would bring his past
into focus.
25. Passage A explains that when writing about the charac-
ter John Huff, Bradbury had:
A. placed John in a town in Arizona, where Bradbury
himself had grown up.
B. included John in stories about a town in Arizona
and in stories about Green Town.
C. moved John to a town other than the town in
which the real-life John Huff had grown up.
D. borrowed John to use as a minor character in
many of his stories.
26. In the first paragraph of Passage B (lines 5263), the
narrator describes John Huff in a manner that:
F. emphasizes John’s physical strength and intelli-
gence, to indicate John’s view of himself.
G. exaggerates John’s characteristics and actions, to
reflect Douglas’s idolization of John.
H. highlights John’s reckless behavior, to show that
Douglas was most fond of John’s rebelliousness.
J. showcases John’s talents, to make clear why both
children and adults admired John.
27. Within Passage B, the image in lines 74–76 functions
figuratively to suggest that:
A. Johns leaving on a stormy night was fitting, given
Douglas’s sadness.
B. Johns disappointment about moving was reflected
in his mood all day.
C. the mood of the day changed dramatically and
irreversibly once John shared his news.
D. the sky in Green Town became cloudy at the
moment John told Douglas he was moving.
28. Both Passage A and Passage B highlight Bradbury’s
use of:
F. a first person omniscient narrator to tell a story.
G. satire and irony to develop characters.
H. allegory to present a complex philosophical
question.
J. sensory details and imaginative description to
convey ideas.
29. Based on Bradbury’s description in Passage A of his
writing process, which of the following methods hypo-
thetically depicts a way Bradbury might have begun to
write the story in Passage B?
A. Taking notes while interviewing old friends after
first deciding to write a story about two boys
B. Forming two characters, determining that he
would like to tell a story about loss, and then
beginning to write a scene
C. Writing down the words train ticket and then
spending an hour writing whatever those words
brought to his mind
D. Outlining the plot of a story about two boys that
would end with one boy leaving on a train
GO ON TO THE NEXT PAGE.
ACT-1572CPRE
33
Questions 2125 ask about Passage A. Questions 26 and 27 ask about Passage B.
Questions 2830 ask about both passages.
GO ON TO THE NEXT PAGE.
ACT-1572CPRE
33
30. Elsewhere in the essay from which Passage A is
adapted, Bradbury writes:
Was there a real boy named John Huff?
There was. And that was truly his name. But
he didn’t go away from me, I went away from
him.
How do these statements apply to both the information
about Bradburys approach as a storyteller provided in
Passage A and the story of John Huff provided in Pas-
sage B?
F. They reveal that Bradbury believed that to surprise
readers is a fiction writer’s most important task.
G. They reinforce that Bradbury used his life experi-
ences to create fiction but also altered those expe-
riences as he pleased.
H. They prove that Bradbury felt such pain over leav-
ing John that he had to reverse events to be able to
write the story.
J. They indicate that Bradbury rarely used his life
experiences to create fiction.
Passage IV
NATURAL SCIENCE: This passage is adapted from the article
“The Jaws That Jump by Adam Summers 2006 by Natural
History Magazine, Inc.).
Recently I was reminded of just how powerful ants
can be when inflicting damage on intruders. A team of
biomechanists has studied the incredibly speedy bite of
a group of Central and South American ants. The team
clocked the bite as the fastest on the planetand dis-
covered that it also gives the ants the unique ability to
jump with their jaws, adding to an impressive array of
already known defenses.
Trap-jaw ants nest in leaf litter, rather than under-
ground or in mounds. There they often feed on well-
armored and elusive prey, including other species of
ants. As they stalk their dinner, the trap-jaws hold their
mandibles wide apart, often cocked open at 180 degrees
or more by a latch mechanism. When minute trigger
hairs on the inner edge of the mandible come in contact
with something, the jaws snap shut at speeds now
known to reach 145 miles per hour. No passerby could
outrace that. The astoundingly high speed gives the
jaws, despite their light weight, enough force to crack
open the armor of most prey and get at the tasty meat
inside.
The key to the jaws speed (and their even more
amazing acceleration) is that the release comes from
stored energy produced by the strong but slow muscles
of the jaw. Think how an archer slowly draws an arrow
in a bowstring against the flex of a bow: nearly all the
energy from the archer’s muscles pours into the flexing
of the bow. When released, the energy stored in the bow
wings the arrow toward its target much faster than the
archer could by throwing the arrow like a javelin. The
biomechanics of energy storage is the domain of Sheila
N. Patek and Joseph E. Baio, both biomechanists at the
University of California, Berkeley. They teamed up
with two ant experts, Brian L. Fisher of the California
Academy of Sciences in San Francisco and Andrew V.
Suarez of the University of Illinois at Urbana-
Champaign, to look at the trap-jaw ant Odontomachus
bauri.
Fisher, Suarez, and other field biologists had
already noted that catching O. bauri was like grabbing
for popping popcornand very hot popcorn at that,
because a painful sting goes with an ant’s trap-jaw bite.
The insects bounced around in a dizzying frenzy and
propelled themselves many times their body length
when biologists or smaller intruders approached them.
Patek and Baio made high-speed video images of their
movements, and discovered that the secret of their self-
propulsion was the well-executed firing of their
mandibles. They also observed that mandibles started to
decelerate before they meetpossibly to avoid self-
inflicted damage. Most important, the ants had two dis-
tinct modes of aerial locomotion.
In the so-called escape jump, an ant orients its
head and jaws perpendicular to the ground, then slams
its face straight down. That triggers the cocked
mandibles to release with a force 400 times the ant’s
body weight, launching the insect ten or more body
lengths nearly straight into the air. The ant doesnt
seem to go in any particular direction, but the jump is
presumably fast and unpredictable enough to help the
insect evade, say, the probing tongue of a lizard. Not
only can the jumping ant gain height and sow confu-
sion, but it may also get to a new vantage point from
which to relaunch an attack.
The second kind of jaw-propelled locomotion is
even more common than escape jumping. If an intruder
enters the ants nest, one of the ants bangs its jaws
against the intruder, which triggers the trap-jaw and
propels the interloper (if small enough) in one direc-
tion, out of the nest, and the ant in the other. Often the
force sends the ant skimming an inch off the ground for
nearly a foot. The attack, for obvious reasons, is known
as the bouncer defense.” In the wild, gangs of defend-
ing ants team up to attack hostile strangers, sending
them head over heels out of the nest.
From an evolutionary point of view, the trap-jaws
are an intriguing story. The ants clearly evolved an
entirely new function, propulsion, for a system that was
already usefulchewing up prey. Several lineages of
trap-jaw ants have independently hit on the tactic of
storing energy in their jaws to penetrate well-defended
prey. In Odontomachus, the horizontal, bouncer-
defense jump could have arisen out of attempts to bite
intruders, but the high, escape jumpwith jaws aimed
directly at the groundmust have arisen from a differ-
ent, perhaps accidental kind of behavior. Such a
serendipitous event would have been a rare instance in
which banging one’s head against the ground got good
results.
5
10
15
20
25
30
35
40
45
50
55
60
65
70
75
80
85
38
39
ACT-1572CPRE
33
31. The primary purpose of the passage is to:
A. provide an overview of the mechanics and key
operations of the jaws of trap-jaw ants.
B. analyze Patek and Baio’s techniques for filming
two defensive maneuvers of trap-jaw ants.
C. compare the jaws of Odontomachus bauri to the
jaws of other species of ants.
D. describe the evolution of the ability of trap-jaw
ants to perform an escape jump.
32. The sentence in lines 7375 and the last sentence of
the passage are examples of the author’s rhetorical
technique of:
F. weaving sarcasm into a mostly casual and playful
article.
G. interjecting a lighthearted tone into a primarily
technical article.
H. integrating a slightly combative tone into an arti-
cle that mostly praises two scientists’ work.
J. incorporating personal anecdotes into an article
that mostly reports data.
33. As it is used in lines 8182, the phrase well-defended
prey most nearly refers to prey that:
A. have a hard outer shell.
B. attack with a lethal bite.
C. travel and attack in groups.
D. move quickly.
34. The passage makes clear that the main source of the
speed of the jaws of the trap-jaw ant is the:
F. ease of movement of the hinge of the jaw.
G. continuous, steady firing of the jaws mandibles.
H. light weight of the jaw in relation to the ant’s body
weight.
J. release of energy stored by muscles of the jaw.
35. The author uses the analogy of trying to grab popcorn
as it pops in order to describe the trap-jaw ants ability
to:
A. generate heat with their jaw movements.
B. move to high ground in order to attack prey.
C. attack intruders by tossing them out of the nest.
D. bounce around frantically when intruders approach.
36. One main purpose of the last paragraph is to suggest
that unlike their bouncer-defense jump, the trap-jaw
ants escape jump may have arisen through:
F. the ants trying and failing to bite intruders.
G. a change in the structure of the mandibles of sev-
eral lineages of ants.
H. an accidental behavior of the ants.
J. the ants experiencing a positive outcome when
they would attack in a large group.
37. As it is used in line 31, the word domain most nearly
means:
A. living space.
B. area of expertise.
C. taxonomic category.
D. local jurisdiction.
38. The passage points to which of the following as a char-
acteristic of trap-jaw ants mandibles that prevents the
ants from harming themselves with their powerful
bite?
F. A hinge prevents the mandibles from snapping
together forcefully.
G. Mandibles with cushioned inner edges provide a
buffer when the mandibles snap shut.
H. A latch mechanism prevents the mandibles from
closing completely.
J. The mandibles begin to decelerate before they
meet.
39. As described in the passage, one benefit of the trap-
jaw ant’s escape jump is that it allows an ant to:
A. land in position to launch a new attack on a
predator.
B. confuse a predator with a quick, sudden sting.
C. signal to other ants using a predictable movement.
D. point itself in whichever direction it chooses to
escape.
40. When a trap-jaw ant uses the bouncer-defense jump
effectively on an intruder, which creature(s), if any,
will be propelled either out of the nest or in another
direction?
F. The intruder only
G. The attacking ant only
H. The attacking ant and the intruder
J. Neither the attacking ant nor the intruder
END OF TEST 3
STOP! DO
NOT TURN THE PAGE UNTIL TOLD TO DO SO.
DO NOT RETURN TO A PREVIOUS TEST.
Passage I
Researchers studied how diet and the ability to smell
food can affect the life span of normal fruit flies (StrainN)
and fruit flies unable to detect many odors (StrainX).
Study 1
Three tubes (Tubes 13), each with 15% sugar yeast
(SY) medium (a diet with 15%sugar and 15%killed yeast),
were prepared. Then, 200virgin female StrainN fruit flies
less than 24 hr old were added to each tube. No additional
substance was added to Tube 1. Additional odors from live
yeast were added to Tube 2, and live yeast was added to
Tube 3. The percent of fruit flies alive was determined
every 5days for 75days (see Figure1).
Figure 1
Study 2
Three tubes (Tubes46), each with 5% SYmedium (a
diet with 5% sugar and 5% killed yeast), were prepared.
Then, 200 virgin female Strain N fruit flies less than 24 hr
old were added to each tube. No additional substance was
added to Tube 4. Additional odors from live yeast were
added to Tube 5, and live yeast was added to Tube 6. The
percent of fruit flies alive was determined every 5 days for
75days (see Figure2).
Figure 2
days
percent alive
100
90
80
70
60
50
40
30
20
10
0
65605550454035302520151050 70 7
5
15% SY medium
15% SY medium + additional odors from live yeast
15% SY medium + live yeast
Key
days
percent alive
100
90
80
70
60
50
40
30
20
10
0
65605550454035302520151050 70 7
5
Key
5% SY medium
5% SY medium + additional odors from live yeast
5% SY medium + live yeast
SCIENCE TEST
35 Minutes—40 Questions
DIRECTIONS: There are several passages in this test.
Each passage is followed by several questions. After
reading a passage, choose the best answer to each
question and fill in the corresponding oval on your
answer document. You may refer to the passages as
often as necessary.
You are NOT permitted to use a calculator on this test.
GO ON TO THE NEXT PAGE.
ACT-1572CPRE
44
40
41
Study 3
Strain N fruit flies were modified to produce Strain X
fruit flies. Strain X fruit flies lack O
83b (a protein
required to detect a wide range of odors); therefore, they
cannot detect many odors. The average life span was deter-
mined for virgin female Strain N and virgin female
Strain X fruit flies fed with various SY media (see
Table1).
Table and gures adapted from Sergiy Libert et al., “Regulation of
Drosophila Life Span by Olfaction and Food-Derived Odors.” ©2007
by the American Association for the Advancement of Science.
1. In which of Studies1 and2 did some of the fruit flies
live for more than 75 days, and what diet were those
fruit flies fed?
A. Study 1; 05% SY medium
B. Study 1; 15% SY medium
C. Study 2; 05% SY medium
D. Study 2; 15% SY medium
2. During Studies1 and2, why did the size of the fruit fly
population in each tube decrease rather than increase?
F. The birthrate was 0, because the initial population
contained only males.
G. The birthrate was 0, because the initial population
contained only virgin females.
H. The death rate was 0, because the initial popula-
tion contained only males.
J. The death rate was 0, because the initial popula-
tion contained only virgin females.
3. Study1 differed from Study 2 in which of the follow-
ing ways?
A. Female fruit flies were tested in Study 1, whereas
male fruit flies were tested in Study2.
B. Male fruit flies were tested in Study 1, whereas
female fruit flies were tested in Study2.
C. The SY medium tested in Study 1 contained a
lower percent of sugar than did the SY medium
tested in Study2.
D. The SY medium tested in Study 1 contained a
higher percent of sugar than did the SY medium
tested in Study2.
4. Suppose that an additional trial in Study 3 had been
performed using a 12% SY medium (a diet with
12% sugar and 12% killed yeast). The average life
span of the Strain X fruit flies in this trial would most
likely have been:
F. less than 55.6 days.
G. between 55.6 days and 58.6 days.
H. between 58.6 days and 61.6 days.
J. greater than 61.6 days.
5. The researchers had predicted that decreasing a fruit
fly’s ability to detect odors would increase its life
span. Are the results of Study 3 consistent with this
prediction?
A. No; for each SY medium tested, the average life
span of Strain X fruit flies was longer than the
average life span of StrainN fruit flies.
B. No; for each SY medium tested, the average life
span of Strain N fruit flies was longer than the
average life span of StrainX fruit flies.
C. Yes; for each SY medium tested, the average life
span of Strain X fruit flies was longer than the
average life span of StrainN fruit flies.
D. Yes; for each SY medium tested, the average life
span of Strain N fruit flies was longer than the
average life span of StrainX fruit flies.
6. Suppose the researchers wanted to determine whether a
defect in the ability to detect odors would change the
life span of fruit flies fed 15% SY medium when live
yeast is added to the diet or when additional odors
from live yeast are added to the diet. Which of the fol-
lowing experiments should be performed?
F. Repeat Study1 except with StrainX fruit flies
G. Repeat Study1 except with StrainN fruit flies
H. Repeat Study2 except with StrainX fruit flies
J. Repeat Study2 except with StrainN fruit flies
7. The results for which 2 tubes should be compared to
determine how a reduced calorie diet affects life span
in the absence of live yeast and additional odors from
live yeast?
A. Tube 1 and Tube 4
B. Tube 1 and Tube 2
C. Tube 2 and Tube 5
D. Tube 5 and Tube 6
Table 1
Strain
SY medium
Average
life span
(days)% sugar
% killed
yeast
Strain N
030, 030, 50.1
050, 050, 50.1
07.5 07.5 43.9
100, 100, 44.8
150, 150, 41.6
Strain X
030, 030, 61.6
050, 050, 62.5
07.5 07.5 58.9
100, 100, 58.6
150, 150, 55.6
GO ON TO THE NEXT PAGE.
ACT-1572CPRE
44
Passage II
In the fall, monarch butterflies (Danaus plexippus) in
eastern North America migrate to Mexico, where they
overwinter in high-altitude forests of oyamel fir (an ever-
green conifer). The butterflies store (accumulate) body
lipids to use as a source of energy at a later time. Consider
the following 3 hypotheses pertaining to when the butter-
flies store lipids and when the energy from the stored lipids
is used, with respect to migration and overwintering.
Hypothesis 1
Monarch butterflies require energy from stored lipids
for migration and during the overwintering period. The
butterflies first store lipids before they begin their migra-
tion. During migration, as stored lipids are converted to
energy, lipid mass continuously decreases. When the but-
terflies reach the overwintering sites, ending their migra-
tion, they must store lipids again before beginning the
overwintering period.
Hypothesis 2
Monarch butterflies require energy from stored lipids
for migration but not during the overwintering period. The
butterflies store lipids before they begin their migration.
During migration, as stored lipids are converted to energy,
lipid mass continuously decreases. Because energy from
stored lipids is not required during the overwintering
period, the butterflies do not store lipids while at the over-
wintering sites.
Hypothesis 3
Monarch butterflies require energy from stored lipids
during the overwintering period but not for migration. The
butterflies do not store lipids before they begin their migra-
tion. Instead, lipids are stored during migration; therefore,
lipid mass continuously increases from the beginning of
migration until the end of migration. The butterflies arrive
at the overwintering sites with enough lipids to provide
themselves with energy during the overwintering period, so
they do not store lipids while at the overwintering sites.
8. Which hypothesis, if any, asserts that monarch butter-
flies store lipids during 2 distinct periods?
F. Hypothesis 1
G. Hypothesis 2
H. Hypothesis 3
J. None of the hypotheses
9. Which hypothesis, if any, asserts that monarch butter-
flies require energy from stored lipids neither for
migration nor during the overwintering period?
A. Hypothesis 1
B. Hypothesis 2
C. Hypothesis 3
D. None of the hypotheses
10. Based on Hypothesis3, which of the following figures
best depicts the change in the lipid mass of a monarch
butterfly from the beginning of migration to the end of
migration?
(Note: In each figure, B represents the beginning of
migration and Erepresents the end of migration.)
F.
G
.
H.
J.
lipid mass
time
B
E
lipid mass
time
B
E
lipid mass
time
B
E
lipid mass
time
B E
GO ON TO THE NEXT PAGE.
ACT-1572CPRE
44
42
43
11. Assume that changes in the body mass of a monarch
butterfly are caused only by changes in the mass of the
butterfly’s stored lipids. The statement The percent of
a monarch butterfly’s body mass that is made up of
lipids is greater at the beginning of migration than at
the end of migration is supported by which of the
hypotheses?
A. Hypothesis 1 only
B. Hypothesis 2 only
C. Hypotheses 1 and 2 only
D. Hypotheses 1, 2, and 3
12. To store lipids, monarch butterflies convert sugar from
nectar they have consumed into lipids. A supporter of
which hypothesis, if any, would be likely to claim that
to ensure the butterflies can store lipids for the over-
wintering period, nectar must be present at the butter-
flies overwintering sites?
F. Hypothesis 1
G. Hypothesis 2
H. Hypothesis 3
J. None of the hypotheses
13. Which of the following statements about lipids in
monarch butterflies is consistent with all 3hypotheses?
A. The butterflies lipid masses do not change during
the overwintering period.
B. The butterflies lipid masses change during
migration.
C. The butterflies use energy from stored lipids
during the overwintering period.
D. The butterflies use energy from stored lipids for
migration.
14. When the monarch butterflies use their stored lipids,
the lipids must be broken down to produce energy-rich
molecules that can be readily used by cells. Which of
the following molecules is produced as a direct result
of the breakdown of the lipids?
F. ATP
G. Starch
H. DNA
J. Amino acids
GO ON TO THE NEXT PAGE.
ACT-1572CPRE
44
Passage III
Greenhouse gases such as methane (CH
4
) warm
Earths climate. Figure 1 shows the concentration of CH
4
in Earth’s atmosphere and the solar radiation intensity at
Earth’s surface for tropical Europe and Asia over the past
250,000 years. As the figure shows, the CH
4
concentration
and the solar radiation intensity have increased and
decreased at the same times over most of this period.
Figure2 shows the same types of data for the same region
over the past 11,000 years. This figure is consistent with
the hypothesis that the greenhouse gases from human
activities may have begun warming Earth’s climate thou-
sands of years earlier than once thought.
GO ON TO THE NEXT PAGE.
ACT-1572CPRE
44
Figure 1
solar radiation intensity
(watts/m
2
)
concentration of CH
4
in
Earth’s atmosphere (ppb*)
540
520
500
480
460
440
900
800
700
600
500
400
300
200
thousands of years ago
250 200 150 100 50 0
(present)
solar radiation
CH
4
concentration
Key
*ppb
=
parts per billion
44
45
Figure 2
Figures adapted from William Ruddiman, Plows, Plagues & Petro-
leum. ©2005 by Princeton University Press.
15. According to Figure 2, the solar radiation intensity
8,000years ago was closest to which of the following?
A. 490watts/m
2
B. 495watts/m
2
C. 500watts/m
2
D. 505watts/m
2
16. According to Figure 2, if the trend in the CH
4
concen-
tration had continued to match the trend in the solar
radiation intensity, the CH
4
concentration at present
would most likely be:
F. less than 550ppb.
G. between 550ppb and 600ppb.
H. between 600ppb and 650ppb.
J. greater than 650ppb.
17. Suppose that whenever the CH
4
concentration
increases, a corresponding, immediate increase in
average global temperature occurs, and that whenever
the CH
4
concentration decreases, a corresponding,
immediate decrease in average global temperature
occurs. Based on Figure 2, which of the following
graphs best represents a plot of average global temper-
ature over the past 11,000years?
18. Based on Figure1, the average solar radiation intensity
over the past 250,000years was closest to which of the
following?
F. 400watts/m
2
G. 440watts/m
2
H. 480watts/m
2
J. 520watts/m
2
19. Onesolar radiation cycle is the time between a maxi-
mum in the solar radiation intensity and the next maxi-
mum in the solar radiation intensity. According to
Figure 1, the average length of a solar radiation cycle
during the past 250,000years was:
A. less than 15,000years.
B. between 15,000years and 35,000years.
C. between 35,000years and 55,000years.
D. greater than 55,000years.
20. Which of the following statements best describes the
primary effect of CH
4
on Earth’s climate?
F. CH
4
gives off visible light to space, cooling
Earth’s climate.
G. CH
4
gives off ultraviolet radiation to space, warm-
ing Earth’s climate.
H. CH
4
absorbs heat as it enters Earth’s atmosphere
from space, cooling Earth’s climate.
J. CH
4
absorbs heat that comes up from Earth’s sur-
face, warming Earth’s climate.
solar radiation intensity (watts/m
2
)
concentration of CH
4
in Earth’s atmosphere (ppb)
thousands of years ago
solar radiation
CH
4
concentration
Key
505
500
495
490
485
480
475
10 5
0
(present)
750
700
650
600
550
500
450
average global
temperature
11
thousands of
years ago
0
average global
temperature
11
thousands of
years ago
0
average global
temperature
11
thousands of
years ago
0
average global
temperature
11
thousands of
years ago
0
A.
B.
C.
D.
GO ON TO THE NEXT PAGE.
ACT-1572CPRE
44
Passage IV
In 2 experiments, a student pulled each of 3 blocks in
a straight line across a flat, horizontal surface.
In Experiment 1, the student measured the pulling
force (the force required to move each block at a constant
speed) and plotted the pulling force, in newtons (N), versus
block mass, in kilograms (kg). The results are shown in
Figure1.
Figure 1
In Experiment 2, the student measured the speed
versus time of a 2.00 kg block, a 2.50 kg block, and a
3.00 kg block as each block was pulled across the surface
with a constant 30 N force. The results are shown in
Figure2.
Figure 2
21. If a block was pulled toward the east, the frictional
force exerted on the block by the surface was directed
toward the:
A. north.
B. south.
C. east.
D. west.
22. Based on Figure 2, what is the order of the 3 blocks,
from the block that required the shortest time to reach
15 m/sec to the block that required the longest time to
reach 15m/sec?
F. 2.00kg block, 2.50kg block, 3.00kg block
G . 2.00kg block, 3.00kg block, 2.50kg block
H . 3.00kg block, 2.00kg block, 2.50kg block
J. 3.00kg block, 2.50kg block, 2.00kg block
23. Based on Figure 2, what was the approximate value of
the acceleration of the 3.00kg block?
A. 00.0m/sec
2
B.
05.0m/sec
2
C. 15.0m/sec
2
D. 20.0m/sec
2
24. Based on Figure 1, the results of Experiment 1 are best
modeled by which of the following equations?
F. Block speed(m/sec)= 0.2
×
time(sec)
G. Block speed(m/sec)= 5.0
×
time(sec)
H. Pulling force(N)= 0.2
×
block mass(kg)
J. Pulling force(N)= 5.0
×
block mass(kg)
0.00
10.00
5.00
20.00
25.00
15.00
0.00
block mass (kg)
1.00
1.50
2.50
3.50
0.50
2.00
3.00
4.5
0
4.00
pulling force (N)
0.00
10.00
5.00
35.00
20.00
25.00
30.00
15.00
0.00
time (sec)
1.00 2.00 3.00
2.50 kg
4.00
speed (m/sec)
3.00 kg
2.00 kg
GO ON TO THE NEXT PAGE.
ACT-1572CPRE
44
46
47
25. At each of the times plotted in Figure 2 (except
0.00sec), as block mass increased, block speed:
A. increased only.
B. decreased only.
C. varied, but with no general trend.
D. remained the same.
26. Based on Figure 1, an applied force of 30.00 N would
most likely have been required to maintain the constant
speed of a block having a mass of:
F. 4.00kg.
G. 5.00kg.
H. 6.00kg.
J. 7.00kg.
GO ON TO THE NEXT PAGE.
ACT-1572CPRE
44
Passage V
A typical acid-base indicator is a compound that will
be one color over a certain lower pH range but will be a
different color over a certain higher pH range. In the small
range between these pH rangesthe transition rangethe
indicator’s color will be an intermediate of its other
2 colors.
Students studied 5 acid-base indicators using colorless
aqueous solutions of different pH and a well plate (a plate
containing a matrix of round depressionswellsthat can
hold small volumes of liquid).
Experiment1
The students added a pH = 0 solution to 5 wells in the
first column of the well plate, then added a pH = 1 solution
to the 5 wells in the next column, and so on, up to pH = 7.
Next, they added a drop of a given indicator (in solution) to
each of the wells in a row, and then repeated this process,
adding a different indicator to each row. The color of the
resulting solution in each well was then recorded in Table 1
(B = blue, G = green, O = orange, P = purple, R = red,
Y= yellow).
Experiment2
Experiment 1 was repeated with solutions that had a
pH of 8 or greater (see Table2).
Experiment3
Students were given 4 solutions (Solutions IIV) of
unknown pH. The well plate was used to test samples of
each solution with 4 of the 5 indicators (see Table3).
Tables adapted from David R. Lide, ed., CRC Handbook of Chem-
istry and Physics, 78th ed. ©1997 by CRC Press LLC.
Table 3
Indicator
Color in Solution:
I II III IV
Metanil yellow YYYO
Resorcin blue BBRR
Curcumin RRYY
Indigo carmine BYBB
Table 1
Indicator
Color in solution with a pH of:
01234567
Metanil yellow RROYYYYY
Resorcin blue RRRRRP PB
Curcumin YYYYYYYY
Hessian bordeaux BBBBBBBB
Indigo carmine BBBBBBBB
Table 2
Indicator
Color in solution with a pH of:
8910 11 12 13 14
Metanil yellow YYYYYYY
Resorcin blue BBBBBBB
Curcumin ORRRRRR
Hessian bordeaux BRRRRRR
Indigo carmine BBBBGYY
GO ON TO THE NEXT PAGE.
ACT-1572CPRE
44
48
49
27. One way Experiment 2 differed from Experiment 3
was that in Experiment2:
A. the solutions to which indicators were added were
of known pH.
B. the solutions to which indicators were added were
of unknown pH.
C. metanil yellow was used.
D. metanil yellow was not used.
28. Based on the description of the well plate and how it
was used, the empty well plate would most likely have
been which of the following colors?
F. Black
G. Blue
H. Red
J. White
29. Based on the results of Experiments1 and2, which of
the following is a possible transition range for
curcumin?
A. pH = 3.9 to pH = 7.3
B. pH = 4.2 to pH = 6.6
C. pH = 7.4 to pH = 8.6
D. pH = 8.4 to pH = 9.5
30. A chemist has 2 solutions, one of pH = 1 and one of
pH = 6. Based on the results of Experiments 1 and 2,
could indigo carmine be used to distinguish between
these solutions?
F. No; indigo carmine is blue at both pH = 1 and
pH= 6.
G. No; indigo carmine is blue at pH = 1 and is yellow
at pH= 6.
H. Yes; indigo carmine is blue at both pH = 1 and
pH= 6.
J. Yes; indigo carmine is blue at pH = 1 and is yellow
at pH= 6.
31. The indicator propyl red has a transition range of
pH = 4.6 to pH = 6.8. If propyl red had been included
in Experiments1 and2, it would have produced results
most similar to those produced by which of the
5 indicators?
A. Metanil yellow
B. Resorcin blue
C. Curcumin
D. Indigo carmine
32. A student claimed that Solution III has a pH of 7.3.
Are the results of Experiments 13 consistent with this
claim?
F. No, because in Solution III metanil yellow was
yellow.
G. No, because in Solution III resorcin blue was red.
H. Yes, because in Solution III metanil yellow was
yellow.
J. Yes, because in Solution III resorcin blue was red.
33. Based on the results of Experiments 13, which of
SolutionsIIV has the lowest pH?
A. Solution I
B. Solution II
C. Solution III
D. Solution IV
GO ON TO THE NEXT PAGE.
ACT-1572CPRE
44
Passage VI
Drilling mud (DM) is a suspension of clay particles in
water. When a well is drilled, DM is injected into the hole
to lubricate the drill. After this use, the DM is brought back
up to the surface and then disposed of by spraying it on
adjacent land areas.
A cover of DM on plants and soil can affect the
albedo (proportion of the total incoming solar radiation
that is reflected from a surface), which in turn can affect
the soil temperature. The effect of a cover of DM on the
albedo and the soil temperature of an unsloped, semiarid
grassland area was studied from July 1 to August 9 of a
particular year.
On June 30, 3 plots (Plots 13), each 10 m by 40 m,
were established in the grassland area. For all the plots, the
types of vegetation present were the same, as was the den-
sity of the vegetation cover. At the center of each plot, a
soil temperature sensor was buried in the soil at a depth of
2.5 cm. An instrument that measures incoming and
reflected solar radiation was suspended 60 cm above the
center of each plot.
An amount of DM equivalent to 40 cubic meters
per hectare (m
3
/ha) was then sprayed evenly on Plot 2.
(One hectare equals 10,000 m
2
.) An amount equivalent to
80 m
3
/ha was sprayed evenly on Plot 3. No DM was
sprayed on Plot1.
For each plot, the albedo was calculated for each
cloudless day during the study period using measurements
of incoming and reflected solar radiation taken at noon on
those days (see Figure1).
Figure 1
For each plot, the sensor recorded the soil temperature
every 5 sec over the study period. From these data, the
average soil temperature of each plot was determined for
each day (see Figure2).
Figure 2
Figures adapted from Franc
s Zvomuya et al., “Surface Albedo and
Soil Heat Flux Changes Following Drilling Mud Application to a
Semiarid, Mixed-Grass Prairie.©2008 by the Soil Science Society
of America.
34. Albedo was measured at noon because that time of day
is when solar radiation reaching the ground is:
F. 100% reflected.
G. 100% absorbed.
H. least intense.
J. most intense.
35. Why was the study designed so that the 3 plots had the
same types of vegetation present and the same density
of vegetation cover? These conditions ensured that any
variations in albedo and soil temperature would most
likely be attributable only to variations among the
plots in the:
A. amount of DM sprayed.
B. type of soil present.
C. plot area.
D. plot slope.
albedo
0.26
0.24
0.22
0.20
0.18
0.16
0.14
June
30
July
5
July
10
July
15
July
20
July
25
July
30
Aug.
4
Aug.
9
Key
Plot 1
Plot 2
Plot 3
dai l y average soil
t e m pe r ature (°C)
28
26
24
22
20
18
June
30
July
5
July
10
July
15
July
20
July
25
July
30
Aug.
4
Aug.
9
Key
Plot 1
Plot 2
Plot 3
GO ON TO THE NEXT PAGE.
ACT-1572CPRE
44
50
51
ACT-1572CPRE
44
36. On one day of the study period, a measurable rainfall
occurred in the study area. The albedo calculated for
the cloudless day just after the rainy day was lower
than the albedo calculated for the cloudless day just
before the rainy day. On which day did a measurable
rainfall most likely occur in the study area?
F. July10
G. July12
H. July26
J. July28
37. For each plot, the number of temperature readings
recorded by the soil temperature sensor every minute
was closest to which of the following?
A. 05
B. 12
C. 50
D. 60
38. According to Figure 1 and the description of the study,
was July20 a cloudless day?
F. No, because albedo data were not collected on that
day.
G. No, because albedo data were collected on that
day.
H. Yes, because albedo data were not collected on
that day.
J. Yes, because albedo data were collected on that
day.
39. According to the results of the study, did the presence
of a cover of DM increase or decrease the albedo, and
did the presence of a cover of DM increase or decrease
the soil temperature?
albedo soil temperature
A. increase increase
B. increase decrease
C. decrease decrease
D. decrease increase
40. Based on Figure 1, on August 3, what percent of
incoming solar radiation was NOT reflected from
Plot2?
F. 20%
G. 40%
H. 60%
J. 80%
END OF TEST 4
STOP! DO NOT RETURN TO ANY OTHER TEST.
[See Note on page 52.]
52
If you plan to take the ACT with writing, sharpen your pencils and
continue with the writing test on page 53.
If you do not plan to take the ACT with writing, skip to page 56 for
instructions on scoring your multiple-choice tests.
53
Directions
This is a test of your writing skills. You will have forty (40) minutes to read the prompt, plan your response,
and write an essay in English. Before you begin working, read all material in this test booklet carefully to
understand exactly what you are being asked to do.
You will write your essay on the lined pages in the answer document provided. Your writing on those pages
will be scored. You may use the unlined pages in this test booklet to plan your essay. Your work on these
pages will not be scored.
Your essay will be evaluated based on the evidence it provides of your ability to:
analyze and evaluate multiple perspectives on a complex issue
state and develop your own perspective on the issue
explain and support your ideas with logical reasoning and detailed examples
clearly and logically organize your ideas in an essay
effectively communicate your ideas in standard written English
Lay your pencil down immediately when time is called.
DO NOT OPEN THIS BOOKLET UNTIL TOLD TO DO SO.
WRITING TEST
BOOKLET
©2015 by ACT, Inc. All rights reserved.
NOTE: This test material is the condential copyrighted property of
ACT, Inc., and may not be copied, reproduced, sold, or otherwise
transferred without the prior express written permission of ACT, Inc.
Violators of ACT’s copyrights are subject to civil and criminal penalties.
PO Box 168
Iowa City, IA 52243-0168
Y o u r S i g n a t u r e : _ _ _ _ _ _ _ _ _ _ _ _ _ _ _ _ _ _ _ _ _ _ _ _ _ _ _ _ _ _ _ _ _ _ _ _ _ _ _ _ _ _ _ _ _ _ _ _ _ _ _ _ _ _ _ _ _ _ _ _ _ _ _ _ _ _
( D o n o t p r i n t . )
P r i n t Y o u r N a m e H e r e : _ _ _ _ _ _ _ _ _ _ _ _ _ _ _ _ _ _ _ _ _ _ _ _ _ _ _ _ _ _ _ _ _ _ _ _ _ _ _ _ _ _ _ _ _ _ _ _ _ _ _ _ _ _ _ _ _ _ _ _
Y o u r D a t e o f B i r t h :
M o n t h D a y Y e a r
Form 15AA51
You must take the multiple-choice tests before you take the writing test.
Practice Writing Test
Form 15AA51
Public Health and Individual Freedom
Most people want to be healthy, and most people want as much freedom as possible to do the things they
icts
Read and carefully consider these perspectives. Each suggests a particular way of thinking about the
conflict between public health and individual freedom.
Perspective One Perspective Two Perspective Three
to achieve the greatest
good for the greatest
Nothing in society is
more valuable than
freedom. Perhaps
The right to avoid health
risks is a freedom,
Essay Task
Write a uni ed, coherent essay in which you evaluate multiple perspectives on the
con ict between public health and individual freedom. In your essay, be sure to:
analyze and evaluate the perspectives given
state and develop your own perspective on the issue
explain the relationship between your perspective and those given
Your perspective may be in full agreement with any of the others, in partial
agreement, or wholly different. Whatever the case, support your ideas with logical
reasoning and detailed, persuasive examples.
54
55
Planning Your Essay
Your work on these prewriting pages will not be scored.
Use the space below and on the back cover to generate ideas and plan your essay. You may wish to
consider the following as you think critically about the task:
Strengths and weaknesses of the three given perspectives
What insights do they offer, and what do they fail to consider?
Why might they be persuasive to others, or why might they fail to persuade?
Your own knowledge, experience, and values
What is your perspective on this issue, and what are its strengths and weaknesses?
How will you support your perspective in your essay?
Note
For your practice essay, you will need scratch paper to plan your essay and four lined sheets
of paper for your response.
On test day, you will receive a test booklet with space to plan your essay and four lined
pages on which to write your response.
Read pages 61–62 for information and instructions on scoring your practice writing test.
56
5
Scoring Your Tests
How to Score the
Multiple-Choice Tests
Follow the instructions below and on the following pages to
score your practice multiple-choice tests and review your
performance.
Raw Scores
The number of questions you answered correctly on each
test and in each subscore area is your raw score. Because
there are many forms of the ACT, each with different
questions, some forms will be slightly easier (and some
slightly harder) than others. A raw score of 67 on one form
of the English test, for example, may be about as difficult to
earn as a raw score of 70 on another form of that test.
To compute your raw scores, check your answers with the
scoring keys on pages 57–58. Count the number of correct
answers for each of the four tests and seven subscore
areas, and enter the number in the blanks provided on
those pages. These numbers are your raw scores on the
tests and subscore areas.
Scale Scores
To adjust for the small differences that occur among
different forms of the ACT, the raw scores for tests and
subscore areas are converted into scale scores. Scale
scores are printed on the reports sent to you and your
college and scholarship choices.
When your raw scores are converted into scale scores, it
becomes possible to compare your scores with those of
examinees who took different test forms. For example, a
scale score of 26 on the English test has the same meaning
regardless of the form of the ACT on which it is based.
To determine the scale scores corresponding to your
raw scores on the practice test, use the tables explaining
procedures used to obtain scale scores from raw scores on
pages 59–60. Table 1 on page 59 shows the raw-to-scale
score conversions for each test, and Table 2 on page 60
shows the raw-to-scale score conversions for the subscore
areas. Because each form of the ACT is unique, each form
has somewhat different conversion tables. Consequently,
these tables provide only approximations of the raw-to-scale
score conversions that would apply if a different form of the
ACT were taken. Therefore, the scale scores obtained from
the practice tests don’t match precisely the scale scores
received from an actual administration of the ACT.
Computing the Composite Score
The Composite score is the average of the four scale scores
in English, mathematics, reading, and science. If you left any
of these tests blank, do not calculate a Composite score.
If you take the ACT with writing, your writing results do not
affect your Composite score.
Comparing Your Scores
Information about comparing your scores on the practice
multiple-choice tests with the scores of recent high school
graduates who took the ACT can be found at
www.actstudent.org.
Your scores and percent at or below are only estimates of the
scores that you will receive during an actual administration
of the ACT. Test scores are only one indicator of your level
of learning. Consider your scores in connection with your
grades, your performance in outside activities, and your
career interests.
ACT College and Career Readiness Standards
The ACT College and Career Readiness Standards
describe the types of skills, strategies, and understandings
you will need to make a successful transition from high
school to college. For English, mathematics, reading,
and science, standards are provided for six score ranges
that reflect the progression andcomplexity of the skills in
each of the academic areas measured by the ACT tests.
Forwriting, standards are provided for five score ranges.
The ACT College and Career Readiness Standards and
benchmark scores for each test can be found at
www.act.org.
Reviewing Your
Performance on the Practice
Multiple-Choice Tests
Consider the following as you review your scores.
Did you run out of time? Reread the information in this
booklet on pacing yourself. You may need to adjust the
way you use your time in responding to the questions.
Did you spend too much time trying to understand the
directions for the tests? The directions for the practice
tests are the same directions that will appear in your test
booklet on test day. Make sure you understand them
before test day.
Review the questions that you missed. Did you select a
response that was an incomplete answer or that did not
directly respond to the question being asked? Try to figure
out what you overlooked in answering the questions.
Did a particular type of question confuse you? Did the
questions you missed come from a particular subscore
area? In reviewing your responses, check to see whether
a particular type of question or a particular subscore
area was more difficult for you.
57
ubscore
Area*
Key EA AG GT
1. D ___
2. H ___
3. E ___
4. F ___
5. E ___
6. H ___
7. E ___
8. H ___
9. A ___
10. K ___
11. C ___
12. K ___
13. B ___
14. H ___
15. B ___
16. H ___
17. D ___
18. F ___
19. D ___
20. F ___
21. B ___
22. H ___
23. A ___
24. H ___
25. B ___
26. G ___
27. E ___
28. H ___
29. C ___
30. G ___
Subscore
Area*
Key EA AG GT
31. D ___
32. K ___
33. B ___
34. H ___
35. D ___
36. J ___
37. A ___
38. F ___
39. B ___
40. F ___
41. E ___
42. K ___
43. D ___
44. G ___
45. D ___
46. J ___
47. B ___
48. G ___
49. A ___
50. F ___
51. E ___
52. H ___
53. B ___
54. K ___
55. E ___
56. K ___
57. A ___
58. K ___
59. E ___
60. J ___
Number Correct (Raw Score) for:
Pre-Alg./Elem. Alg. (EA) Subscore Area _______
(27)
Inter. Alg./Coord. Geo. (AG) Subscore Area _______
(19)
Plane Geo./Trig. (GT) Subscore Area _______
(14)
Total Number Correct for Math Test _______
(EA + AG + GT) (60)
*EA = Pre-Algebra/Elementary Algebra
AG = Intermediate Algebra/Coordinate Geometry
GT = Plane Geometry/Trigonometry 1572CPRE
Scoring Keys for the ACT Practice Tests
Use the scoring key for each test to score your answer document for the multiple-choice tests. Mark a “1” in the
blank for each question you answered correctly. Add up the numbers in each subscore area and enter the total
number correct for each subscore area in the blanks provided. Also enter the total number correct for each test in
the blanks provided. The total number correct for each test is the su
m of the number correct in each subscore area.
Test 1: EnglishScoring Key Test 2: Mathematics—Scoring Key
Subscore
Area*
Key UM RH
1. A ___
2. J ___
3. D ___
4. H ___
5. B ___
6. J ___
7. B ___
8. J ___
9. C ___
10. J ___
11. B ___
12. G ___
13. A ___
14. H ___
15. D ___
16. H ___
17. B ___
18. J ___
19. C ___
20. H ___
21. A ___
22. F ___
23. B ___
24. H ___
25. C ___
26. F ___
27. D ___
28. G ___
29. A ___
30. F ___
31. B ___
32. H ___
33. B ___
34. H ___
35. A ___
36. F ___
37. A ___
38. F ___
Subscore
Area*
Key UM RH
39. C ___
40. J ___
41. C ___
42. G ___
43. D ___
44. J ___
45. C ___
46. J ___
47. D ___
48. H ___
49. B ___
50. F ___
51. D ___
52. J ___
53. A ___
54. F ___
55. B ___
56. G ___
57. C ___
58. F ___
59. D ___
60. G ___
61. C ___
62. G ___
63. D ___
64. F ___
65. C ___
66. H ___
67. D ___
68. G ___
69. D ___
70. J ___
71. A ___
72. F ___
73. B ___
74. H ___
75. D ___
Number Correct (Raw Score) for:
Usage/Mechanics (UM) Subscore Area _______
(40)
Rhetorical Skills (RH) Subscore Area _______
(35)
Total Number Correct for English Test _______
(UM + RH) (75)
*UM = Usage/Mechanics
RH = Rhetorical Skills 1572CPRE
57
5858
est 3: Reading—Scoring Key
Number Correct (Raw Score) for:
Social Studies/Sciences (SS) _______
Subscore Area (20)
Arts/Literature (AL) Subscore Area _______
(20)
Total Number Correct for Reading Test _______
(SS + AL) (40)
Test 4: Science—Scoring Key
Number Correct (Raw Score) for:
Total Number Correct for Science Test _______
(40)
Subscore
Area*
Key SS AL
1. A ___
2. J ___
3. C ___
4. H ___
5. B ___
6. J ___
7. A ___
8. H ___
9. B ___
10. J ___
11. A ___
12. J ___
13. C ___
14. J ___
15. B ___
16. H ___
17. B ___
18. F ___
19. D ___
20. F ___
Subscore
Area*
Key SS AL
21. A ___
22. G ___
23. D ___
24. J ___
25. C ___
26. G ___
27. C ___
28. J ___
29. C ___
30. G ___
31. A ___
32. G ___
33. A ___
34. J ___
35. D ___
36. H ___
37. B ___
38. J ___
39. A ___
40. H ___
*SS = Social Studies/Sciences
AL = Arts/Literature 1572CPRE
Key
1. C ___
2. G ___
3. D ___
4. G ___
5. C ___
6. F ___
7. A ___
8. F ___
9. D ___
10. J ___
11. C ___
12. F ___
13. B ___
14. F ___
15. C ___
16. F ___
17. B ___
18. H ___
19. B ___
20. J ___
Key
21. D ___
22. F ___
23. B ___
24. J ___
25. B ___
26. H ___
27. A ___
28. J ___
29. C ___
30. F ___
31. B ___
32. G ___
33. D ___
34. J ___
35. A ___
36. H ___
37. B ___
38. F ___
39. D ___
40. J ___
1572CPRE
59
TABLE 1
Expl anation of Procedures Used to Obtain Scale Scores from Raw Scores
On each of the four multiple-choice tests on which you
marked any responses, the total number of correct
responses yields a raw score. Use the table below to convert
your raw scores to scale scores. For each test, locate and
circle your raw score or the range of raw scores that includes
it in the table below. Then, read across
to either outside
column of the table and circle the scale score that
corresponds to that raw score. As you determine your scale
scores, enter them in the blanks provided on the right. The
highest possible scale score for each test is 36. The lowest
possible scale score for any test on which you marked any
responses is 1.
Next, compute the Composite score by averaging the four
scale scores. To do this, ad
d your four scale scores and
divide the sum by 4. If the resulting number ends in a
fraction, round it to the nearest whole number. (Round down
any fraction less than one-half; round up any fraction that is
one-half or more.) Enter this number in the blank. This is your
Composite score. The highest possible Composite score is
36. The lowest possible Composite score is 1.
ACT Test 1572CPRE Your Scale Score
English ________
Mathematics ________
Reading ________
Science ________
Sum of scores ________
Composite score (sum ÷ 4) ________
Writing ________
NOTE: If you left a test completely blank and marked no
items, do not list a scale score for that test. If any test was
completely blank, do not calculate a Composite score.
To calculate your writing score, use the rubric on
pages 61–62.
1572CPRE
Scale
Score
Raw Scores
Scale
Score
Test 1
English
Test 2
Mathematics
Test 3
Reading
T
est 4
Science Writing
36
35
34
33
32
31
30
29
28
27
26
25
24
23
22
21
20
19
18
17
16
15
14
13
12
11
10
9
8
7
6
5
4
3
2
1
75
72-74
71
70
68-69
67
66
65
63-64
62
60-61
58-59
56-57
53-55
51-52
48-50
45-47
43-44
41-42
39-40
36-38
32-35
29-31
27-28
25-26
23-24
20-22
18-19
15-17
12-14
10-11
8-9
6-7
4-5
2-3
0-1
60
58-59
57
55-56
54
52-53
50-51
48-49
45-47
43-44
40-42
38-39
36-37
34-35
32-33
30-31
29
27-28
24-26
21-23
17-20
13-16
11-12
08-10
7
5-6
4
3
2
1
0
40
39
38
37
35-36
34
33
32
31
30
29
28
27
25-26
24
22-23
21
19-2
0
18
17
15-16
14
12-13
11
09-10
8
6-7
5
4
3
2
1
0
40
39
38
37
36
35
34
33
32
30-31
28-29
26-27
24-25
22-23
21
19-20
17-18
16
14-15
13
12
11
10
9
8
7
5-6
4
3
2
1
0
47-48
46
44-45
42-43
41
40
38-39
37
35-36
34
33
32
31
29-30
28
26-27
25
24
23
21-22
20
18-19
17
16
14-15
13
12
10-11
9
8
36
35
34
33
32
31
30
29
28
27
26
25
24
23
22
21
20
19
18
17
16
15
14
13
12
11
10
9
8
7
6
5
4
3
2
1
Explanation of Procedures Used to Obtain
Scale Subscores from Raw Scores
For each of the seven subscore areas, the total number of correct
responses yields a raw score. Use the table below to convert your raw
scores to scale subscores. For each of the seven subscore areas,
locate and circle either the raw score or the range of raw scores that
includes it in the table below. Then, read across to either
outside
column of the table and circle the scale subscore that corresponds to
that raw score. As you determine your scale subscores, enter them in
the blanks provided on the right. The highest possible scale subscore
is 18. The lowest possible scale subscore is 1.
If you left a test completely blank and marked no responses, do not list
any scale subscores for that test.
ACT Test 1572CPRE Your Scale Subscore
English
Usage/Mechanics ________
Rhetorical Skills ________
Mathematics
Pre-Algebra/Elementary Algebra ________
Algebra/Coord. Geometry ________
Plane Geometry/Trigonometry ________
Reading
Social Studies/Sciences ________
Arts/Literature ________
TABLE 2
1572CPRE
cale
Subscore
Raw Scores
Scale
Subscore
Test 1 English
est 2 Mathematics Test 3 Reading
Usage/
Mechanics
Rhetorical
Skil
ls
Pre-Algebra/
Elem. Algebra
Algebra/
Coord. Geometry
Plane Geometry/
Trigonometry
Social Studies/
Sciences
Arts/
Literature
18
17
16
15
14
13
12
11
10
9
8
7
6
5
4
3
2
1
38-40
36-37
35
33-34
32
31
29-30
27-28
25-26
23-24
20-22
18-19
16-17
13-15
10-12
8-9
5-7
0-4
35
34
32-33
31
29-30
27-28
25-26
22-24
20-21
17-19
15-16
13-14
11-12
09-10
7-8
5-6
2-4
0-1
26-27
24-25
22-23
21
20
18-19
17
15-16
14
13
11-12
08-10
6-7
5
3-4
2
1
0
19
18
17
15-16
14
12-13
10-11
9
7-8
6
4-5
3
2
1
0
14
12-13
11
10
9
8
7
6
5
4
3
2
1
0
20
19
17-18
16
15
13-14
12
11
09-10
8
7
5-6
4
3
2
1
0
19-20
18
17
16
15
14
13
12
11
10
9
8
6-7
5
3-4
2
1
0
18
17
16
15
14
13
12
11
10
9
8
7
6
5
4
3
2
1
60
61
How to Score the Writing Test
It is difficult to be objective about one’s own work. However,
it is to your advantage to read your own writing critically, as
doing so can help you grow as a writer and as a reader. It may
also be helpful for you to give your practice essay to another
reader, such as a classmate, parent, or teacher. To rate your
essay, you and your reader(s) should review the guidelines
an
d sample essays at w w w
act student.org and then use the
scoring rubric below to assign your practice essay a score of
1 (low) through 6 (high) in each of the four writing domains
(Ideas and Analysis, Development and Support, Organization,
Language Use).
Scoring Rubric (below)
The rubric presents the standards by which your essay will be
evaluated. Readers will use this rubric to assign your essay
four uni
que scores, one per writing domain. To score your
essay, determine which scorepoint, in each domain, best
describes the features of your writing. Because each domain
receives its own score, the four scores you assign need not be
identical. For example, you may find that your essay exhibits
stronger skill in organization than in the development of ideas.
In this case, you may determine that your essay sh
ould
receive a higher score in Organization than in Development
and Support.
The ACT Writing Test Scoring Rubric
Ideas and Analysis Development and Support Organization Language Use
Score 6:
Responses at
this scorepoint
demonstrate
effective skill
in writing an
argumentative
essay.
The writer generates an
argument that critically
engages with multiple
perspectives on the given
issue. The argument’s
thesis reflects nuance and
precision in thought and
purpose. The argument
establishes and employs an
insightful context for
analysis of the issue and its
perspectives. The analysis
examines implications,
complexities and tensions,
and/or underlying values
and assumptions.
Development of ideas and
support for claims deepen
insight and broaden
context. An integrated line
of skillful reasoning and
illustration effectively
conveys the significance of
the argument. Qualifications
and complications enrich
and bolster ideas and
analysis.
The response exhibits a
skillful organizational
strategy. The response is
unified by a controlling idea
or purpose, and a logical
progression of ideas
increases the effectiveness
of the writer’s argument.
Transitions between and
within paragraphs
strengthen the relationships
among ideas.
The use of language
enhances the argument.
Word choice is skillful and
precise. Sentence
structures are consistently
varied and clear. Stylistic
and register choices,
including voice and tone,
are strategic and effective.
While a few minor errors in
grammar, usage, and
mechanics may be present,
they do not impede
understanding.
Score 5:
Responses at
this scorepoint
demonstrate
well-developed
skill in
writing an
argumentative
essay.
The writer generates an
argument that productively
engages with multiple
perspectives on the given
issue. The argument’s
thesis reflects precision in
thought and purpose. The
argument establishes and
employs a thoughtful
context for analysis of the
issue and its perspectives.
The analysis addresses
implications, complexities
and tensions, and/or
underlying values and
assumptions.
Development of ideas and
support for claims deepen
understanding. A mostly
integrated line of purposeful
reasoning and illustration
capably conveys the
significance of the
argument. Qualifications
and complications enrich
ideas and analysis.
The response exhibits a
productive organizational
strategy. The response is
mostly unified by a
controlling idea or purpose,
and a logical sequencing of
ideas contributes to the
effectiveness of the
argument. Transitions
between and within
paragraphs consistently
clarify the relationships
among ideas.
The use of language works
in service of the argument.
Word choice is precise.
Sentence structures are
clear and varied often.
Stylistic and register
choices, including voice
and tone, are purposeful
and productive. While minor
errors in grammar, usage,
and mechanics may be
present, they do not impede
understanding.
Score 4:
Responses at
this scorepoint
demonstrate
adequate skill
in writing an
argumentative
essay.
The writer generates an
argument that engages with
multiple perspectives on the
given issue. The argument’s
thesis reflects clarity in
thought and purpose. The
argument establishes and
employs a relevant context
for analysis of the issue and
its perspectives. The
analysis recognizes
implications, complexities
and tensions, and/or
underlying values and
assumptions.
Development of ideas and
support for claims clarify
meaning and purpose.
Lines of clear reasoning
and illustration adequately
convey the significance of
the argument. Qualifications
and complications extend
ideas and analysis.
The response exhibits a
clear organizational
strategy. The overall shape
of the response reflects an
emergent controlling idea or
purpose. Ideas are logically
grouped and sequenced.
Transitions between and
within paragraphs clarify
the relationships among
ideas.
The use of language
conveys the argument with
clarity. Word choice is
adequate and sometimes
precise. Sentence
structures are clear and
demonstrate some variety.
Stylistic and register
choices, including voice
and tone, are appropriate
for the rhetorical purpose.
While errors in grammar,
usage, and mechanics are
present, they rarely impede
understanding.
The ACT Writing Test Scoring Rubric
Ideas and Analysis Development and Support Organization Language Use
Score 3:
Responses at
this scorepoint
demonstrate
some
developing skill
in writing an
argumentative
essay.
The writer generates an
argument that responds to
multiple perspectives on the
given issue. The argument’s
thesis reflects some clarity
in thought and purpose.
The argument establishes a
limited or tangential context
for analysis of the issue and
its perspectives. Analysis is
simplistic or somewhat
unclear.
Development of ideas and
support for claims are
mostly relevant but are
overly general or simplistic.
Reasoning and illustration
largely clarify the argument
but may be somewhat
repetitious or imprecise.
The response exhibits a
basic organizational
structure. The response
largely coheres, with most
ideas logically grouped.
Transitions between and
within paragraphs
sometimes clarify the
relationships among ideas.
The use of language is
basic and only somewhat
clear. Word choice is
general and occasionally
imprecise. Sentence
structures are usually clear
but show little variety.
Stylistic and register
choices, including voice
and tone, are not always
appropriate for the
rhetorical purpose.
Distracting errors in
grammar, usage, and
mechanics may be present,
but they generally do not
impede understanding.
Score 2:
Responses at
this scorepoint
demonstrate
weak or
inconsistent
skill in
writing an
argumentative
essay.
The writer generates an
argument that weakly
responds to multiple
perspectives on the given
issue. The argument’s
thesis, if evident, reflects
little clarity in thought and
purpose. Attempts at
analysis are incomplete,
largely irrelevant, or consist
primarily of restatement of
the issue and its
perspectives.
Development of ideas and
support for claims are
weak, confused, or
disjointed. Reasoning and
illustration are inadequate,
illogical, or circular, and fail
to fully clarify the argument.
The response exhibits a
rudimentary organizational
structure. Grouping of ideas
is inconsistent and often
unclear. Transitions
between and within
paragraphs are misleading
or poorly formed.
The use of language is
inconsistent and often
unclear. Word choice is
rudimentary and frequently
imprecise. Sentence
structures are sometimes
unclear. Stylistic and
register choices, including
voice and tone, are
inconsistent and are not
always appropriate for the
rhetorical purpose.
Distracting errors in
grammar, usage, and
mechanics are present, and
they sometimes impede
understanding.
Score 1:
Responses at
this scorepoint
demonstrate
little or no skill
in writing an
argumentative
essay.
The writer fails to generate
an argument that responds
intelligibly to the task. The
writer’s intentions are
difficult to discern. Attempts
at analysis are unclear or
irrelevant.
Ideas lack development,
and claims lack support.
Reasoning and illustration
are unclear, incoherent, or
largely absent.
The response does not
exhibit an organizational
structure. There is little
grouping of ideas. When
present, transitional devices
fail to connect ideas.
The use of language fails
to demonstrate skill in
responding to the task.
Word choice is imprecise
and often difficult to
comprehend. Sentence
structures are often unclear.
Stylistic and register
choices are difficult to
identify. Errors in grammar,
usage, and mechanics are
pervasive and often impede
understanding.
Calculating Your Writing Subject Score
Complete these steps to calculate your Writing Subject Score (1–36 scale).
1. Locate the four domain scores (1–6) and enter them in the first column below. Double each score and enter in the Domain
Score column to the right.
2. Enter the sum of the second-column scores here ______. This is your raw score (value between 8 and 48).
3. Use Table 1 on page 59to find the
scaled Writing Subject Score that corresponds to your raw score.
Domain
Score
Ideas and Analysis ____ x 2 = ____
Development and Support ____ x 2 = ____
Organization ____ x 2 = ____
Language Use and Conventions ____ x 2 = ____
62
63
You may wish to remove this sample answer document from the booklet to use in a practice test session for the four multiple-choice tests.
|
BA C D
\
\
1
0
\
\
\
\
\
\
\
\
\
\
1
2
3
4
5
6
7
8
9
0
\
\
\
\
\
\
\
\
\
\
1
2
3
4
5
6
7
8
9
0
\
\
\
\
\
\
\
\
\
\
1
2
3
4
5
6
7
8
9
0
\
\
\
\
\
\
\
\
\
\
1
2
3
4
5
6
7
8
9
0
\
\
\
\
\
\
\
\
\
\
1
2
3
4
5
6
7
8
9
0
\
\
\
\
\
\
\
\
\
\
1
2
3
4
5
6
7
8
9
0
\
\
\
\
\
\
\
\
\
\
1
2
3
4
5
6
7
8
9
0
\
\
\
\
\
\
\
\
\
\
1
2
3
4
5
6
7
8
9
0
\
\
\
\
\
\
\
\
\
\
1
2
3
4
5
6
7
8
9
0
\
\
\
\
\
\
\
\
\
\
\
\
\
\
\
\
\
\
\
\
\
\
\
\
\
\
\
\
\
\
\
\
\
\
\
\
\
\
\
\
\
\
\
1
2
3
0
\
\
\
\
\
\
\
\
\
\
1
2
3
4
5
6
7
8
9
0
\
\
\
\
\
\
\
\
\
\
1
2
3
4
5
6
7
8
9
0
\
\
\
\
\
\
\
\
\
\
1
2
3
4
5
6
7
8
9
0
A
B
C
D
E
F
G
H
I
J
K
L
M
N
O
P
Q
R
S
T
U
V
W
X
Y
Z
\
\
\
\
\
\
\
\
\
\
\
\
\
\
\
\
\
\
\
\
\
\
\
\
\
\
\
A
B
C
D
E
F
G
H
I
J
K
L
M
N
O
P
Q
R
S
T
U
V
W
X
Y
Z
\
\
\
\
\
\
\
\
\
\
\
\
\
\
\
\
\
\
\
\
\
\
\
\
\
\
\
A
B
C
D
E
F
G
H
I
J
K
L
M
N
O
P
Q
R
S
T
U
V
W
X
Y
Z
\
\
\
\
\
\
\
\
\
\
\
\
\
\
\
\
\
\
\
\
\
\
\
\
\
\
\
A
B
C
D
E
F
G
H
I
J
K
L
M
N
O
P
Q
R
S
T
U
V
W
X
Y
Z
\
\
\
\
\
\
\
\
\
\
\
\
\
\
\
\
\
\
\
\
\
\
\
\
\
\
\
A
B
C
D
E
F
G
H
I
J
K
L
M
N
O
P
Q
R
S
T
U
V
W
X
Y
Z
EXAMINEE STATEMENT, CERTIFICATION, AND SIGNATURE
1.
Read t he following Statement: B y submi tting t his answer sheet, I agree to comply wi th and be bound by t he Terms and Conditions:
Testing Rules and Policies for the ACT
®
provided in the AC T r egist rat ion material s for this as sess ment, incl uding t hose concer ning test
secur it y, s core canc ell ation, exam inee remedi es, ar bitr ation, and c onsent to the proces sing of my personal ly ident if ying infor mati on,
incl uding the col lect ion, use, tr ansfer and dis clos ure of informat ion as descr ibed in the ACT Pr ivacy Pol icy (ww w. act. org/ pr ivac y. html ).
International Examinees: By my s ignat ure I am al so prov iding my c onsent to AC T to trans fer my pers onally i denti fy ing i nfor mat ion
to the U nited St ates to ACT, or a third par t y s er vi ce prov ider for process ing, w here it wil l be subjec t to us e and di scl osure under the
laws of the U nited States. I acknow ledge and agree t hat i t may al so be ac cess ibl e to law enforcem ent and nat ional s ecur ity
authorit ies in the United St ates.
I understand t hat AC T owns the ass essm ent quest ions and res ponses and affir m that I wil l not s hare any asses sment ques ti ons or
res ponses wit h anyone by any for m of comm unicat ion before, dur ing, or after the asses sment adm inis trat ion. I unders tand t hat
assum ing anyone el ses i dentit y to take t his ass essm ent i s s tr ict ly prohibi ted and m ay v iol ate t he l aw and s ubject me to legal penal ti es.
2. Copy the Certification shown below (only the text in italic s) on the li nes provi ded. Wr ite in your nor mal handwrit ing.
Certification: I agree to the Statement above and certify that I am the person whose name and address appear on this answer sheet.
Your Si gnature Todays Date
PO BOX 168, IOWA CITY, IOWA 52243-0168
© 2015 by AC T, Inc . A ll rights res er ved. 21668 011 215 160 Rev 1IM-(A)194993-001:654321Printed in U.S.A.
NAME, MAILING ADDRESS, AND TELEPHONE
(Please print.)
USE A SOFT LEAD NO. 2 PENCIL ONLY.
(Do NOT use a mechanical pencil, ink, ballpoint, correction fluid, or felt-tip pen.)
Last Name Fi rs t Name MI (Middl e Init i al)
House Number & S treet (Apt. No.); or PO B ox & No.; or RR & No.
Cit y State/Provi nce ZI P/ Pos tal Code
Area Code Number Countr y
DATE OF
BIRTH
Mont h Day Year
Jan.
F eb.
Marc h
Apr il
May
June
J uly
A ug.
Sept .
Oc t.
N o v .
Dec .
The ACT
®
2015–2016 Answer Sheet (No Writing)
MATCH NUMBER
PAGE 1
ACT, Inc.—Confidential Restricted when data present
ALL examinees must complete block A please print.
Blocks B, C, and D ar e requi red for all exam inees. Find
the M ATCHING I NF ORMATI ON on your t icket. E nter it E XAC TLY
the s ame way, even if any of the infor mat ion i s mi ss ing or
incorrec t. Fi ll in t he cor responding ovals . I f you do not com plete
these bl ocks to match y our previous infor mat ion EX AC TLY, your
scores wi ll be delayed up to 8 weeks.
/
Do NOT
mark in
this shaded
area.
MATCH
NAME
(Fi rst 5 letters
of l ast name)
»
Cut Here
\
\
\
\
\
\
\
\
\
\
1
2
3
4
5
6
7
8
9
0
\
\
\
\
\
\
\
\
\
\
1
2
3
4
5
6
7
8
9
0
\
\
\
\
\
\
\
\
\
\
1
2
3
4
5
6
7
8
9
0
\
\
\
\
\
\
\
\
\
\
1
2
3
4
5
6
7
8
9
0
\
\
\
\
\
\
\
\
\
\
1
2
3
4
5
6
7
8
9
0
\
\
\
\
\
\
\
\
\
\
1
2
3
4
5
6
7
8
9
0
\
\
\
\
\
\
\
\
\
\
\
\
\
\
\
\
\
\
\
\
\
\
\
\
\
\
\
\
\
\
\
\
\
\
\
\
\
\
\
\
\
\
\
\
\
\
\
\
\
\
\
\
\
\
\
\
\
\
\
\
\\\\
\
\
\
\
\
\
\
\
\
\
\
\
\
\
\
\
\
\
\
\
\
\
\
\
\
\
\
\
\
\
\
\
\
\
\
\
\
\
\
\
\
\
\
\
\
\
\
\
\
\
\
\
A
F
A
F
A
F
A
F
A
F
A
F
A
B
G
B
G
B
G
B
G
B
G
B
G
B
C
H
C
H
C
H
C
H
C
H
C
H
C
D
J
D
J
D
J
D
J
D
J
D
J
D
\
\
\
\
\
\
\
\
\
\
\
\
\
\
\
\
\
\
\
\
\
\
\
\
\
\
\
\
\
\
\
\
\
\
\
\
\
\
\
\
A
F
A
F
A
F
A
F
A
F
B
G
B
G
B
G
B
G
B
G
C
H
C
H
C
H
C
H
C
H
D
J
D
J
D
J
D
J
D
J
\
\
\
\
\
\
\
\
\
\
E
K
E
K
E
K
E
K
E
K
\
\
\
\
\
\
\
\
\
\
\
\
\
\
\
\
\
\
\
\
\
\
\
\
\
\
\
\
\
\
\
\
\
\
\
\
\
\
\
\
A
F
A
F
A
F
A
F
A
F
B
G
B
G
B
G
B
G
B
G
C
H
C
H
C
H
C
H
C
H
D
J
D
J
D
J
D
J
D
J
\
\
\
\
\
\
\
\
\
\
E
K
E
K
E
K
E
K
E
K
\
\
\
\
\
\
\
\
\
\
\
\
\
\
\
\
\
\
\
\
\
\
\
\
\
\
\
\
\
\
\
\
\
\
\
\
\
\
\
\
A
F
A
F
A
F
A
F
A
F
B
G
B
G
B
G
B
G
B
G
C
H
C
H
C
H
C
H
C
H
D
J
D
J
D
J
D
J
D
J
\
\
\
\
\
\
\
\
\
\
E
K
E
K
E
K
E
K
E
K
\
\
\
\
\
\
\
\
\
\
\
\
\
\
\
\
\
\
\
\
\
\
\
\
\
\
\
\
\
\
\
\
\
\
\
\
\
\
\
\
A
F
A
F
A
F
A
F
A
F
B
G
B
G
B
G
B
G
B
G
C
H
C
H
C
H
C
H
C
H
D
J
D
J
D
J
D
J
D
J
\
\
\
\
\
\
\
\
\
\
E
K
E
K
E
K
E
K
E
K
\
\
\
\
\
\
\
\
\
\
\
\
\
\
\
\
\
\
\
\
\
\
\
\
\
\
\
\
\
\
\
\
\
\
\
\
\
\
\
\
A
F
A
F
A
F
A
F
A
F
B
G
B
G
B
G
B
G
B
G
C
H
C
H
C
H
C
H
C
H
D
J
D
J
D
J
D
J
D
J
\
\
\
\
\
\
\
\
\
\
E
K
E
K
E
K
E
K
E
K
\
\
\
\
\
\
\
\
\
\
\
\
\
\
\
\
\
\
\
\
\
\
\
\
\
\
\
\
\
\
\
\
\
\
\
\
\
\
\
\
A
F
A
F
A
F
A
F
A
F
B
G
B
G
B
G
B
G
B
G
C
H
C
H
C
H
C
H
C
H
D
J
D
J
D
J
D
J
D
J
\
\
\
\
\
\
\
\
\
\
E
K
E
K
E
K
E
K
E
K
\
\
\
\
\
\
\
\
\
\
\
\
\
\
\
\
\
\
\
\
\
\
\
\
\
\
\
\
\
\
\
\
\
\
\
\
\
\
\
\
\
\
\
\
\
\
\
\
\
\
\
\
F
A
F
A
F
A
F
A
F
A
F
A
F
G
B
G
B
G
B
G
B
G
B
G
B
G
H
C
H
C
H
C
H
C
H
C
H
C
H
J
D
J
D
J
D
J
D
J
D
J
D
J
\
\
\
\
\
\
\
\
\
\
\
\
\
\
\
\
\
\
\
\
\
\
\
\
\
\
\
\
A
F
A
F
A
F
A
B
G
B
G
B
G
B
C
H
C
H
C
H
C
D
J
D
J
D
J
D
\
\
\
\
\
\
\
\
\
\
\
\
\
\
\
\
\
\
\
\
\
\
\
\
\
\
\
\
F
A
F
A
F
A
F
G
B
G
B
G
B
G
H
C
H
C
H
C
H
J
D
J
D
J
D
J
\
\
\
\
\
\
\
\
\
\
\
\
\
\
\
\
\
\
\
\
\
\
\
\
\
\
\
\
A
F
A
F
A
F
A
B
G
B
G
B
G
B
C
H
C
H
C
H
C
D
J
D
J
D
J
D
\
\
\
\
\
\
\
\
\
\
\
\
\
\
\
\
\
\
\
\
\
\
\
\
\
\
\
\
F
A
F
A
F
A
F
G
B
G
B
G
B
G
H
C
H
C
H
C
H
J
D
J
D
J
D
J
\
\
\
\
\
\
\
\
\
\
\
\
\
\
\
\
\
\
\
\
\
\
\
\
\
\
\
\
A
F
A
F
A
F
A
B
G
B
G
B
G
B
C
H
C
H
C
H
C
D
J
D
J
D
J
D
\
\
\
\
\
\
\
\
\
\
\
\
\
\
\
\
\
\
\
\
F
A
F
A
F
G
B
G
B
G
H
C
H
C
H
J
D
J
D
J
\
\
\
\
\
\
\
\
\
\
\
\
\
\
\
\
\
\
\
\
\
\
\
\
\
\
\
\
A
F
A
F
A
F
A
B
G
B
G
B
G
B
C
H
C
H
C
H
C
D
J
D
J
D
J
D
\
\
\
\
\
\
\
\
\
\
\
\
\
\
\
\
\
\
\
\
\
\
\
\
\
\
\
\
F
A
F
A
F
A
F
G
B
G
B
G
B
G
H
C
H
C
H
C
H
J
D
J
D
J
D
J
\
\
\
\
\
\
\
\
\
\
\
\
\
\
\
\
\
\
\
\
\
\
\
\
\
\
\
\
A
F
A
F
A
F
A
B
G
B
G
B
G
B
C
H
C
H
C
H
C
D
J
D
J
D
J
D
\
\
\
\
\
\
\
\
\
\
\
\
\
\
\
\
\
\
\
\
\
\
\
\
\
\
\
\
F
A
F
A
F
A
F
G
B
G
B
G
B
G
H
C
H
C
H
C
H
J
D
J
D
J
D
J
\
\
\
\
\
\
\
\
\
\
\
\
\
\
\
\
\
\
\
\
\
\
\
\
\
\
\
\
A
F
A
F
A
F
A
B
G
B
G
B
G
B
C
H
C
H
C
H
C
D
J
D
J
D
J
D
\
\
\
\
\
\
\
\
\
\
\
\
\
\
\
\
\
\
\
\
F
A
F
A
F
G
B
G
B
G
H
C
H
C
H
J
D
J
D
J
\
\
\
\
\
\
\
\
\
\
\
\
\
\
\
\
\
\
\
\
\
\
\
\
\
\
\
\
\
\
\
\
\
\
\
\
\
\
\
\
\
\
\
\
\
\
\
\
\
\
\
\
A
F
A
F
A
F
A
F
A
F
A
F
A
B
G
B
G
B
G
B
G
B
G
B
G
B
C
H
C
H
C
H
C
H
C
H
C
H
C
D
J
D
J
D
J
D
J
D
J
D
J
D
\
\
\
\
\
\
\
\
\
\
\
\
\
\
\
\
\
\
\
\
\
\
\
\
\
\
\
\
\
\
\
\
\
\
\
\
\
\
\
\
\
\
\
\
\
\
\
\
\
\
\
\
F
A
F
A
F
A
F
A
F
A
F
A
F
G
B
G
B
G
B
G
B
G
B
G
B
G
H
C
H
C
H
C
H
C
H
C
H
C
H
J
D
J
D
J
D
J
D
J
D
J
D
J
\
\
\
\
\
\
\
\
\
\
\
\
\
\
\
\
\
\
\
\
\
\
\
\
\
\
\
\
\
\
\
\
\
\
\
\
\
\
\
\
\
\
\
\
\
\
\
\
\
\
\
\
A
F
A
F
A
F
A
F
A
F
A
F
A
B
G
B
G
B
G
B
G
B
G
B
G
B
C
H
C
H
C
H
C
H
C
H
C
H
C
D
J
D
J
D
J
D
J
D
J
D
J
D
\
\
\
\
\
\
\
\
\
\
\
\
\
\
\
\
\
\
\
\
\
\
\
\
\
\
\
\
\
\
\
\
\
\
\
\
\
\
\
\
F
A
F
A
F
A
F
A
F
A
G
B
G
B
G
B
G
B
G
B
H
C
H
C
H
C
H
C
H
C
J
D
J
D
J
D
J
D
J
D
\
\
\
\
\
\
\
\
\
\
\
\
\
\
\
\
\
\
\
\
\
\
\
\
\
\
\
\
\
\
Inc orr ec t mar k s:
Over lapping mar k:
Cros s- out mar k :
Sm udged eras ure:
Mar k is too li ght:
64F
68E
69C
69E
70D
70E
71B
71F
72A
72D
72E
72F
73A
73D
73E
73F
73H
74A
75A
75B
75C
75D
75E
75F
75G
75H
75R
75S
76A
96H
96Y
98H
98J
98K
98L
98N
98P
98Q
98R
98T
PAGE 2
BOOKLET NUMBER
TEST 1
TEST 2
TEST 3
TEST 4
ACT STUDENT REVIEW: The test administrator wil l give you instructions for completing this s ec t i on.
FORM
Pr int your
3-charac ter
Test Form i n
the boxes
above and
fi ll in the
corres ponding
oval at the
r ight .
Marking Directions: Mark only one oval for
each question. Fill in response completely.
Erase errors cleanly without smudging.
Do NOT use these incorrect or bad marks.
Correct mark:
BE SURE TO FILL IN THE CORRECT FORM OVAL.
1
2
3
4
5
6
7
8
9
10
11
12
13
14
15
16
17
18
19
20
21
22
23
24
25
26
27
28
29
30
31
32
33
34
35
36
37
38
39
40
41
42
43
44
45
46
47
48
49
50
51
52
53
54
55
56
57
58
59
60
61
62
63
64
65
66
67
68
69
70
71
72
73
74
75
1
2
3
4
5
6
7
8
9
10
11
12
13
14
15
16
17
18
19
20
21
22
23
24
25
26
27
28
29
30
31
32
33
34
35
36
37
38
39
40
41
42
43
44
45
46
47
48
49
50
51
52
53
54
55
56
57
58
59
60
1
2
3
4
5
6
7
8
9
10
11
12
13
14
1
2
3
4
5
6
7
8
9
10
11
12
13
14
15
16
17
18
19
20
21
22
23
24
25
26
27
28
29
30
31
32
33
34
35
36
37
38
39
40
15
16
17
18
19
20
21
22
23
24
25
26
27
28
29
30
31
32
33
34
35
36
37
38
39
40
Student Review: Your res ponses to these i tems w il l as si st ACT
and your test c enter i n pr ov idi ng the bes t pos si ble conditions for
tes ti ng and pl anning for t he f uture. Fi ll i n t he oval i ndic ati ng your
res ponse to each item pr inted on t he back of your t est bookl et.
1
2
3
4
5
6
7
8
9
10
11
12
13
14
15
Yes
No Yes N o Yes No
PRE
64
3832
*080192160* Rev 1